.
. centre de ressources dilemmes et doutes le visage humain de mathématiques Qui sommes-nous Problème de mois activités de promotion babillard
Centrale des maths - centraledesmaths.uregina.ca
Dilemmes & doutes
« D & D »
. .
topic card  

Sujet:

numbers

liste de
sujets
. .
nouvelle recherche

478 articles trouvés pour ce sujet.
 
Page
1/1
2^(2^1954) est-il premier? 2005-10-04
Un eleve pose la question :
2(21945)+1 est-il premier?
Claude Tardif lui répond.
Avec les chiffres 1 3 4 6... 2005-04-14
Ambre et Miel pose la question :
avec les chiffres 1 3 4 6 utilisé une seule fois chacun soit en div multipl addit ousoustr je dois avoir 24 comme résultat
Claude Tardif lui répond.
Combien y aura t il de 9 dans la réponse? 2000-06-17
Marie pose la question :
J'ai lu une énigme mais je n'ai pas la réponse."si l'on divise 123 456 789 par 999 999 999, combien y aura t il de 9 dans la réponse?"
Claude Tardif lui répond.
The next rational number 2022-01-14
Sophia pose la question :
what comes after 3 1/2?
Penny Nom lui répond.
Simplifying a complex expression 2021-12-15
Grace pose la question :
Write in the form. a+bi, -6-7i+8i^2+4i3-5i^4
Penny Nom lui répond.
Multiplying and adding two numbers 2021-02-06
Yvonne pose la question :
Two numbers multiplied to get -13 and added to get -12
Harley Weston lui répond.
Simplify 2021-02-05
Leslie pose la question :
To simplify improper fractions, do you convert it to a mixed number or leave as an improper fraction?

My daughter had a quiz in which she was supposed to simplify her answers. So, for example, on one question the answer she got was 11/10 and on another question she came up with 17/12. The teacher marked these as incorrect because they were mixed numbers. Her teacher wrote on her paper, "**name**, I even gave this back to you and told you to go through and simplify your answers!" We (her father and I) feel that the teacher wanted her to convert her answers, not simplify them but I haven't been able to find a definitive answer online. So, does simplifying improper fractions mean converting them to mixed numbers? Thank you for your help!


Penny Nom lui répond.
0^(a+bi) 2020-12-03
Douglas pose la question :
I realize raising 0^a = 0 if a>0 and undefined if a<=0.

If have read that 0^bi is undefined for all b.

What I don't understand is why 0^(a+bi) = 0 if a and b are not equal to zero.

Is this purely by definition or is there a logical reason why this is the case?
(I have taken Complex Analysis, so have a fairly good understanding of complex numbers.)

Penny Nom lui répond.
Five rational numbers between -4 and -3 2020-07-28
Priyanshi pose la question :
Five rational numbers between -4 and 3
Penny Nom lui répond.
Three numbers have a sum of 45 2020-05-06
Jayden pose la question :
Three numbers have a sum of 45. The greatest of the 3 numbers is 2 greater than the least number. What are the numbers? *
Penny Nom lui répond.
Four digit numbers starting with 6 2020-04-22
Judy pose la question :
I need to generate a list of 4 digit numbers using 0-9 where the first digit is 6 and no numbers can be repeated
Penny Nom lui répond.
Six-digit telephone numbers 2020-03-12
Lulamile pose la question :
What is the probability that a six-digit telephone number has no repeated digits? The telephone number cannot begin with a zero
Penny Nom lui répond.
Investigating y = (-2)^x 2020-01-13
Gonzalo pose la question :
This is not precisely a maths question, but it is formulated based on my maths curiosity. Fidgetting with my new graphic calculator, I started graphing things and had the idea to graph $y=(-2)^x.$ The result surprised me, and I thought a little bit about it, stored it on the back of my brain, and promised myself to look deeper into it someday.
Harley Weston lui répond.
Two digit numbers 2019-12-03
Lacy pose la question :
Using the digits zero through nine, how many two-digit numbers can be formed,which do not have a four or five?
Penny Nom lui répond.
Even 5 digit palindromes 2019-10-24
daniel pose la question :
How many palindromes of 5 digits are even?
Penny Nom lui répond.
Six digit numbers from 4 digits 2019-07-30
Tab pose la question :
How many six digit combinations can be made from the numbers zero, two, five, eight? With repetition of numbers
Penny Nom lui répond.
Two numbers 2019-05-03
fds pose la question :
The difference of two numbers is 2, and their sum is 4. What are the two numbers?
Penny Nom lui répond.
A word problem 2019-02-24
Briana pose la question :
When 4 times the number increased by 30, the answer is the same as when 70 decreased by the number. What was my number?
Harley Weston lui répond.
Adding and multiplying two numbers 2019-02-13
katie pose la question :
what 2 numbers add to get 7 and multiply to get 2?
Penny Nom lui répond.
Numbering pages 2019-01-20
chungus pose la question :
It takes 852 digits to number the pages of a book consecutively. How many pages are there?
Penny Nom lui répond.
Four digit numbers from 0, 1, 2 and 3 2018-11-30
A parent pose la question :
how many four digit numbers can be formed using nos.0,1,2,3 if repetition is allowed?
Penny Nom lui répond.
Compatible numbers 2018-09-19
Jessica pose la question :
Hello, I'm having trouble helping my daughter she is in 5th grade and I tell you her math is so much different then the math I learned in school and I been out of school for quite some time so it's a little hard for me.
Anyway her math is very confusing for me they are saying use compatible numbers to find two estimates.
I have no idea what that even mean I tried using Google but still don't get it . The problem is 12 divided by 478 using compatible numbers to find two estimates.... please help

Penny Nom lui répond.
The sum and difference of two numbers 2018-09-17
Feni pose la question :
The sum of two numbers is 4. The difference of the two numbers is 2. What are the two numbers?
Penny Nom lui répond.
The sum and difference of two numbers 2018-08-30
Saranya pose la question :
Hi,

The question is to find the numbers from sum and difference.
The Sum = 14 and the difference = 2

I found in this web site some useful answers. but my kid is going second grade only. She don't know how to do a division or multiplication.
So can you please explain on how to find the answer using only the Addition and Subtraction.

Thanks in advance.

Penny Nom lui répond.
Three consecutive whole numbers 2018-06-22
whaha pose la question :
the sum of three consecutive whole numbers divided by 4 is equal to 6. find the numbers.
Penny Nom lui répond.
The product of two numbers 2018-06-19
amie pose la question :
the product of two numbers is 108.when the larger number is divided by the smaller number the quotient is 3. What are the number.tnx
Penny Nom lui répond.
An equation with rational coefficients 2018-05-26
Anagh pose la question :
If x and y are rational number and
(x+y) + (x - 2y)√2 = 2x -y + (x-y-1) √5

Then find the value of x and y.

Penny Nom lui répond.
Gauss' Addition of whole numbers. 2018-04-30
Brad pose la question :
I found this on your site. Question: what is the sum of the first 100 whole numbers?? Is there a different formula if the numbers begin at a number other than one? For example What is the series I want to add is goes from 7 - 53?
Harley Weston lui répond.
The sum and difference of two numbers 2018-03-13
samima pose la question :
Two numbers have a difference of 0.85 and the sum 1.What are the numbers?
Penny Nom lui répond.
Some 5 number sets 2018-03-02
Sarah pose la question :
Hello! My question is:
If I have 5 columns of numbers, and I want to create all possible combinations (or is it permutations?) How can I do that? For example all numbers in each column may not appear in any other column. so:
column 1: 1,2,3,4,5
column 2: 6,7,8,9,10
column 3: 11,12,13,14,15
column 4: 16,17,18,19,20
column 5: 21,22,23,24,25

how may I determine all the possible ways I can get sets of 5 out if this? Thanks!

Penny Nom lui répond.
Simplify 1/2^i 2018-01-28
Deepak pose la question :
How do I simplify this complex equation:

z=1/(2^i)

Penny Nom lui répond.
The sum and difference of two numbers 2018-01-25
Ali pose la question :

Hello,
I was looking at the original question that was posted and answered by
your team:
http://mathcentral.uregina.ca/QQ/database/QQ.09.07/s/donna1.html

I used the 7 and 4 as example and not looking for 39 per original
question:

A=7
B=4

7+ 4 =11 and 7-4 = 3

We end up with 11 +3 = 14

A= 14/2 give use 7

B = how do you solve for B or 4 with out knowing anything about 3 or any other numbers ?

Thank you
Ali


Penny Nom lui répond.
The decimal form of a fraction 2018-01-12
Tom pose la question :
Prove that the decimal representation of the quotient of 2 integers must repeat (if it does not terminate).
Penny Nom lui répond.
Compatible numbers 2018-01-02
Natalie pose la question :
I'm in 4th grade and two weeks ago my class learned how to use compatible numbers. I was REALLY confused but I didn't want to say I didn't understand because I just wanted to go home. Tomorrow we have the Topic 7 Test and I'm stuck on a problem that requires using compatible numbers. My mom apparently hasn't heard of compatible numbers and she can't really help me too much. Here's the problem : The local library places new books in a section with 31 shelves. Each shelf fits 18 books. Use compatible numbers to estimate the number of books that the library can fit on the shelves.
Penny Nom lui répond.
Retiring athletes numbers 2017-12-18
Robin pose la question :
I am wondering if you can explain in layman terms how to figure out how many different ways the numbers 00-99 can be arranged? The discussion that sparked this question revolved around the practice of retiring athletes numbers and how many actual number combinations there are. So if I were to use 00, 01, 02 03, 04, 05, 06, 07, 08, 09, 1, 2, 3, 4, 5, 6, 7, 8, 9, 10, 11, ....99. I say there are only 109 different numbers. I'm being told I'm wrong that there are way more than that. Please help. Thank you, Robin
Penny Nom lui répond.
27000001 2017-10-09
Tulashiram pose la question :
If a× b =27000001, then what is the value of a & b ?
Penny Nom lui répond.
Two digit numbers that contain 5 2017-08-11
Irish pose la question :
How many two digit numbers contain the digit 5?
Penny Nom lui répond.
Some 4 digit numbers 2017-07-24
Camille pose la question :
I need all of the 4 digit combinations that starts with number 3 and digits can't be repeated
Penny Nom lui répond.
Three consecutive natural numbers 2017-06-11
Anita pose la question :
Three consecutive natural numbers are such that the square of the middle number exceeds the difference of the square of the other two by 60. Find the numbers.
Penny Nom lui répond.
4 digit codes with repeating digits 2017-06-01
Morgan pose la question :
what are all possible 4 digit code repeating numbers? cuz i know that you have one website on it but, it doesn't repeat 2 numbers in a code.
Harley lui répond.
Five rational numbers between -2 and -1 2017-05-31
Prince pose la question :
Five rational numbers between -2 and -1
Penny Nom lui répond.
Rational numbers between -2 and -3 2017-04-08
Medha pose la question :
Find 12 rational numbers between-2 and -3
Penny Nom lui répond.
The sum and difference of two numbers 2017-03-20
Olivia pose la question :
The sum of two numbers is 87 and their difference is 29. What are the two numbers
Penny Nom lui répond.
Write these numbers from greatest to least 2016-12-22
lsaiah pose la question :
write the following numbers from greatest to least

3 1/2, 7/3, 80%, 2.5, 0.94

Penny Nom lui répond.
x^2 = -16 2016-12-12
A student pose la question :
x to the second power = -16

what number solves the equation?

Penny Nom lui répond.
Four digit numbers 2016-09-08
ning pose la question :
0000-9999 there is ten thousand 4-digit numbers. How to find the number of ways that 4 digit are the same? How to prove it by using formula?
Penny Nom lui répond.
The modulus of a complex number 2016-07-29
Cheyenne pose la question :
There's a question on my Summer Assignment I cant figure out. Here it is:

Find the absolute Value of the complex number. -5i

Please help?

Penny Nom lui répond.
Writing numbers in the form a/b 2016-07-17
Edward pose la question :
How to write these numbers in form of a/b: -7, 0.175, -3.25, 37/10, -16, 1/3, 2.04
Penny Nom lui répond.
A word problem 2016-02-27
KRK pose la question :
amrutha thinks of a number, multiplies it by 5 & 6, devides the result by 3 and takes away the number first thought of to get an answer 36. find out the number
Penny Nom lui répond.
Complex numbers in standard form 2016-01-15
Michael pose la question :
express the following complex numbers in standard form (2+3i)+(5-2i)
Penny Nom lui répond.
Sue's favourite number 2016-01-12
Maria pose la question :
This is my son's homework. He is 7. Can you help?

Sue has a favourite number.
It is an even number.
It has 3 digits.
The digits add up to 15.

Find 10 numbers that could be Sue's number?

favourite number lui répond.
Is 22/7 bigger or smaller than 3.14? 2015-11-06
Natalie pose la question :
Is 22/7 bigger or smaller than 3.14?
Harley Weston lui répond.
An example of an irrational number 2015-10-22
Allison pose la question :
It ask me to find an example of an irrational number less than -5 and I don't understand what the difference from a rational number and an irrational number besides the fact that a rational number can be repeated and shown in a simple fraction and an irrational number can't be written in a simple fraction.
Can you help me?

Penny Nom lui répond.
Square roots 2015-09-21
mariana pose la question :
I have read various articles on how to find the square root of irrational numbers and every article out there seems to be very confusing.
i read you answer to LUKOW about irrational numbers and i am still quite confused. Say i want to find the square root of 326. i know that it is between 18 and 19 because 18 is the square root of 324 and 19 is the square root of 361 im just very confused about the rest of the process. Please help! ( if possible i would appreciate two examples. thanks)

Penny Nom lui répond.
Mutually prime composite numbers 2015-01-09
nivedita pose la question :
Two composite number that is mutually prime?
Penny Nom lui répond.
The units digit of a prime 2015-01-08
Patricia pose la question :
Every prime number greater than 10 has a digit in the ones place that is included in which set of numbers below 1 3 7 9 or 1 3 7 or 0 2 4 5 6 8 or 1 3 5 9
Walter Whiteley lui répond.
Missing digits in a phone number 2015-01-07
Janelle pose la question :
I had a friend try and play a game on me, regarding phone numbers, so I only know the area code , and the last four digits of the phone number. Which means, there are 3 digits that I need to figure out all the possibilities to.
Penny Nom lui répond.
The sum of two numbers is 52 2014-09-01
Blake pose la question :
The sum of two numbers is 52 and the difference is 10. What are the numbers? i used to be real good at this stuff?
Penny Nom lui répond.
How do I make fractions into decimals? 2014-05-31
jay pose la question :
Hi,
I wanted to know how do I make fractions into decimals example 1 3/8?

Penny Nom lui répond.
The combination of 6 numbers from 1 to 355 that gives us 360 2014-05-29
Ion pose la question :
Hi,
My name is Ion and I am trying to find the number of combinations of 6 numbers (angles) that would sum to 360 (degrees).

Thank you!

Robert Dawson lui répond.
Three digit numbers 2014-05-15
Mudassir pose la question :
how many 3 digits numbers which at least one digit is 2 ?
Penny Nom lui répond.
Three consecutive whole numbers 2014-03-13
Abhishek pose la question :
Three consecutive whole numbers are such that if they be divided by 5,3 and 4 respectively, the sum of the quotient is 40. What are the numbers?
Chris Fisher lui répond.
Four digit phone numbers 2014-02-25
Tom pose la question :
Hi, I'm an online math teacher, working on lessons for my students.

This question is in their book and I can't do it...! Help!

It has to do with phone numbers, and in this problem, we are only dealing with the last 4 digits of the number.

It asks how many possible numbers we have if at least one digit repeats in the last 4 digits of the number.

Penny Nom lui répond.
7 digit phone numbers 2013-12-08
Sean pose la question :
Hi I was wondering how you would calculate how may 7 digit phone numbers there are with only odd digits?
Penny Nom lui répond.
Adding mixed numbers 2013-11-20
Kathy pose la question :
1 3/4 + 1 2/3= ?

5 1/2 - 2 5/6= ?

Penny Nom lui répond.
Two, 2-digit numbers 2013-09-24
Tony pose la question :
Write a problem that has two 2-digit numbers.

solve your problem.

Penny Nom lui répond.
What is the value of |2((i)^(1/2))|? 2013-07-22
Delilah pose la question :
What is the value of |2((i)^(1/2))| ?
i.e. absolute value of 2 multiplied by square root of i.

Penny Nom lui répond.
Two numbers whose difference is 16 2013-06-08
Johan pose la question :
Two numbers whose difference is 16 and whose sum is 120
Penny Nom lui répond.
Write a rule for the sequence 2013-02-20
Angelia pose la question :
Write a rule for the sequence. Then, find the unknown term. 1 3/8, 1 ¾, 2 1/8, _______, 2 7/8 Think: The pattern is increasing. Add 1/6 to find the next term. Rule: _______________
Penny Nom lui répond.
2 1/4 x 1/8 x 1 3/4 x 12 4/9 x 3 2012-12-21
Tony pose la question :
Ok my question

2 1/4 x 1/8 x 1 3/4 x 12 4/9 x 3

I just can't figure out the question ?

Penny Nom lui répond.
Some 4 digit numbers 2012-10-25
samira pose la question :
How many 4-digit numbers are there in which the first and last digits are the same?
Penny Nom lui répond.
Some 6 digit numbers 2012-10-23
Mason pose la question :
How many different 6 digit numbers can you make using the digits 1 ,2 5, 6, 7, and 9? How many of these six-digit numbers are divisible by 6?
Penny Nom lui répond.
Six digit numbers 2012-09-11
Fiona pose la question :
using the numerals 0, 2, 3, 5, 7, 9, make as many six-digit numbers as you can. Rearrange them into ascending order
Robert Dawson lui répond.
Kirkman schoolgirl problem 2012-08-18
Anthony pose la question :
need to have the number from 1-15 in 6 rounds of 3 numbers without the numbers be repeating in the same row . eg round 1
1 2 3
4 5 6
7 8 9
10 11 12
13 14 15

2nd round
1 4 7
10 13 2
5 8 11
14 3 6
9 12 15
Chris Fisher lui répond.

A number puzzle 2012-04-07
Kenya pose la question :
my number is less than 25
my number of tiles will make only one rectangle
my number is odd
my number is a factor of 36

Penny Nom lui répond.
Square numbers 2012-03-17
david pose la question :
How many square numbers are there between 0 to 100?
Penny Nom lui répond.
Properties of real numbers applied to subsets 2012-02-01
Mark pose la question :
Hello - The questions that I have for you is do the properties of real numbers (such as the associative, commutative, identity, inverse, and distributive law) apply to ALL the subsets of real numbers? In other words, do all those properties work for the Natural Numbers? The Whole Numbers? And so on and so forth. I understand that they are all real numbers, but for instance: the identity is whenever you add zero to a number, you get that number back. But does that work with, say, with only the odd numbers? Zero isn't odd so can that property actually apply to JUST the odd numbers? Any consideration would be greatly appreciated!
Robert Dawson lui répond.
Triangular numbers that are square 2011-12-27
Luke pose la question :
The “TnS” is a positive integer which is both Triangular number and Square number. For example, 36 is a “TnS” number since 36=1+2+3+4+5+6+7+8 (Triangular number) and 36=6x6 (Square number). What is the next “TnS” which is greater than 36?
Penny Nom lui répond.
The number of digits in a number base m 2011-12-22
Jash pose la question :
Assume there is a number system of base m.

The one property of this system is: If 2 numbers written in this system, which have 'a' and 'b' as the number of digits are multiplied, then the product of the 2 numbers will have a number of digits which is a function f(a,b).

In other words, as long as the number of digits of the 2 numbers are constant, the number of digits of their product is a constant.

Find m.

Robert Dawson lui répond.
Extended real numbers 2011-12-12
Justin pose la question :
Hi there, I was wondering does +infinity=+infinity in the extended real number system? Basically, I was wondering does +infinity=+infinity since -infinity and any extended real number (except +infinity) are less than +infinity?

Sincerely,

Justin

Robert Dawson lui répond.
Three consecutive numbers 2011-11-29
Michael pose la question :
Find three consecutive numbers such that the second number squared is equal to the first and third added together
Penny Nom lui répond.
15 pigs and 6 pens 2011-11-16
Mike pose la question :
I have 15 pigs and 6 pens, I have to put them in odd numbers, how do I do it.
Chris Fisher lui répond.
I started with Euler's identity and manipulated it 2011-11-14
anonymous pose la question :
I started with Euler's identity and manipulated it
e^i*pi=-1
e^-i*pi=(-1)^-1
e^-i*pi=-1
e^-i*i*pi=(-1)^i
e^--pi=(-1)^i
e^pi=(-1)^i
type it in in a calculator and you get e^pi=23.1406926... and (-1)^i=0.0432139183... What did I do wrong?

Robert Dawson lui répond.
Twice an even number 2011-10-10
Ron pose la question :
The number of dollars that Tracy has in her pocket is an odd number. Hal gave Tracy $2 more than she already had in her pocket. The number of dollars Tracy now has cannot be _________.
a) $60 b) $62 c) $64 d) 68

Penny Nom lui répond.
3-digit, daily numbers 2011-09-29
Margaret pose la question :
I need the list/group for a raffle that would contain the combinations for a 3 digit (000-999) daily number. There would be 100 tickets sold with 10 3-digit numbers from each numerical group beginning from 000's, 100's 200's, 300's,...900's. No duplicates/replacement. What would be the list of possible combinations of each group of 100 tickets, keeping each different group/list of combinations in separate blocks of 100 tickets without duplication. So I could use 1 list of 100 this year and the next list of combos next year and so forth. Please help!

Example: 000,197,245,367,445,569,618,777,842,964

What would be the possible lists of each 100 tickets? Thank you

Penny Nom lui répond.
One factor array 2011-09-16
Hana pose la question :
hi my brother a 5th grade have a question. this is what it say "i have only one factor array, and i am between two numbers 12 and 16. what number am i?"
Penny Nom lui répond.
Ben's favourite numbers 2011-08-03
Luke pose la question :
Ben challenges you to guess his favourite numbers by using the following three clues: -Each number is a four-digit odd number. -Each is a palindrome -The digits of each number add up to 10 -None of the numbers are divisible by 5
Robert Dawson lui répond.
Two whole numbers 2011-05-11
yolanda pose la question :
The sum of two whole numbers is 12.If the sum of the squares of those numbers is 74,what are the two numbers?
Penny Nom lui répond.
5 digit numbers using the digits 5 and 2 2011-04-27
sissy pose la question :
I need a list of all possible 5 digit combinations using 5 and 2 numbers can repeat
Penny Nom lui répond.
Decreasing numbers 2011-04-13
Hadi pose la question :
A number is called a "decreasing number" if each digit in the number is less than the digit to its left. For example, 87420 is a deceasing number. How many five-digit decreasing numbers are there?
Claude Tardif lui répond.
8 3/8 - 6 1/4 2011-03-21
lenora pose la question :
explain an error pattern in each of the following. 8 3/8 - 6 1/4 = 2 2/4
Penny Nom lui répond.
Three consecutive odd numbers 2011-03-11
George pose la question :
Write an equation three consecutive odd numbers whose sum is 75
Penny Nom lui répond.
Three consecutive numbers 2011-01-31
parth pose la question :
name 3 consecutive numbers,each less then a 100,the smallest # is divisible by 6 the next is divisibel by 5 and the largest divisible by 4
Penny Nom lui répond.
A mixed number 2011-01-30
Joyce pose la question :
express the fraction as a mixed or whole number 94/5
Penny Nom lui répond.
Find all the roots 2010-12-02
gagan pose la question :
find all the roots of z^5-3z^4+2z^3+z^2-3z+2
Stephen La Rocque and Penny Nom lui répond.
Ordering mixed numbers 2010-11-23
kate pose la question :
what are the mixed numbers 5 7/9, 5 1/2, 5 11/18. put least to greatest?
Penny Nom lui répond.
i^i 2010-11-21
trale pose la question :
Can we use e^ix=cosx+isinx for finding i^i like that: x= pi/2 => e^(ipi/2)=0+i then [e^(ipi/2)]^i=i^i.then we find i^i= 0,207879576.... is it true? can we give value for x for free?thank you.
Harley Weston lui répond.
What is the largest prime number? 2010-11-20
vadali pose la question :
what is the largest prime number?
Penny Nom lui répond.
Four-digit numbers using 0,3,4,5,6,and 7 2010-11-12
Katy pose la question :
Using only the digits 0,3,4,5,6,and 7, how many distinct four-digit numbers exist that are greater than 4002 and less than 6732?
Stephen La Rocque lui répond.
z^5 - 3z^4 + 2z^3 + z^2 - 3z + 2 2010-11-06
Kumar pose la question :
would you please solve this problem, related to complex numbers.

Find all the roots of :

z^5 - 3z^4 + 2z^3 + z^2 - 3z + 2

Robert Dawson and Penny Nom lui répond.
A Squared Number That's Negative 2010-09-22
David pose la question :
What is the only number that when it's squared becomes negative?
Stephen La Rocque lui répond.
777777 times 111111 2010-07-07
Chew pose la question :
What is 777777 times 111111 without using a calculator?
Chris Fisher lui répond.
Graphical Representation of Complex Numbers 2010-06-08
Anas pose la question :
why do we write complex number a+ib as (a,b)?
Janice Cotcher lui répond.
11 trillion divided by 309,418,000 2010-06-05
Dion pose la question :
I am struggling to calculate what 11 trillion divided by 309,418,000 is.
Penny Nom lui répond.
A quadratic equation with imaginary numbers 2010-06-03
Alissa pose la question :
I am solving a quadratic equation and I got this far;
(x-4+i)(x-4-i)=0
but how do I add the imaginary numbers i know you multiply x by x and then add -4 + -4 but what do you do with the i's?

Penny Nom lui répond.
Four digit numbers 2010-05-22
tyeisha pose la question :
Whats all the four digit numbers you can come up with using numbers 0-9
Tyler Wood lui répond.
The sum of twice a number and 6 is 8 2010-05-12
MAE pose la question :
THE SUM OF TWICE A NUMBER AND 6 IS 8 . FIND THE NUMBER
Penny Nom lui répond.
Odd numbers with no repeated digits 2010-05-01
jameka pose la question :
How many odd numbers can be written from the set {2,3,4,5,6} if no digits may be used more than once?
Penny Nom lui répond.
A champagne tower 2010-04-16
gabriel pose la question :
thanks for having something like this up online. My question is how do i build a Champagne Tower that has 290 glasses and 15 stories high.
Please help me out. this is for a clarity function.

Robert Dawson lui répond.
Body measurements 2010-04-06
Amirul pose la question :
Recently I'm proposing my research question to my teacher for my extended essay. I'm an IB student. My research question is regarding the estimation of human in buying trousers through reference of neck. What does the relation between the diameter of the neck and the diameter of the waist? I want to see how far does the estimation theory is true for different type of people with different BMI(body mass index)..

But teacher said that it is golden ratio...so nothing interesting... =( really??? But i search on net.... state that my idea seems do not have any relation with the golden ratio so far..... i just want ask you... am I able to perform in my extended essay if i continue with this research question??

Robert Dawson lui répond.
A champagne pyramid 2010-03-22
Kathy pose la question :
I have 680 champagne glasses and my pyramid can only be 15 levels high. How many glasses need to be on each level? Is there a formula I can use?
Robert Dawson lui répond.
7 odd numbers 2010-02-19
mike pose la question :
Hello there, Problem: find 7 odd numbers who sum is equal to 30. Hint*My professor said it's possible to add 7 odd numbers and get 30 for an answer! and he said we can use Negative and rational numbers but no decimal or fractions. So is there anyway of solving this? Please get back to ASAP! thanks.
Harley Weston and Tyler Wood lui répond.
4-digit positive numbers 2010-02-15
marjon pose la question :
Find the sum of all the 4-digit positive numbers with no zero digit.
Penny Nom lui répond.
Mixed numbers and multiplication 2010-02-08
Nick pose la question :
Can you explain how this works, I have been going at it for hours and have a test friday and just cant get my head around the question
3 1\4 * 1/7 * 8

I have to express in lowest terms

Penny Nom lui répond.
Three digit odd numbers 2010-02-04
Rebecca pose la question :
Question from rebecca, a student:

how many three digit odd numbers can be formed using the digits 1,2,3,4,5,6 without using any digit more than once?

Tyler Wood lui répond.
1/a + 1/b + 1/c < 1 2010-02-01
Mrityun pose la question :
suppose a,b and c are natural numbers such that 1/a + 1/b + 1/c < 1. Prove that

1/a + 1/b + 1/c < = 41/42.

Penny Nom and Claude Tardif lui répond.
Mixed numbers 2010-01-13
Linh pose la question :
Perform the indicated operations:
6 1/4 + (- 2/3) x 1 1/2

Penny Nom lui répond.
A proof involving real numbers 2010-01-11
Amper pose la question :
Let a,b is an element of real numbers, and suppose that for every x>0 we have a is lesser than or equal to b+x.
(a) Show that a is lesser than or equal to b.
(b) Show that it does not follow that a is lesser than b.
i'm feeling bad of having no idea with this, hope i you can help me. GRACIAS!!

Penny Nom lui répond.
A set of 5 numbers 2010-01-10
Marco pose la question :
I need to generate in an Excel or other table all possible combinations for a set of 5 numbers with the following criteria:

1.- Numbers can repeat.
2.- Order does not matter.
3.- Numbers may be from 0-9
4.- The sum of all numbers is equal or less than 31.
5.- The sum of all numbers is equal or greater than 10.
6.- One of the numbers is 2.
7.- One of the numbers is 5.
8.- One of the numbers is 7.

Marco

Penny Nom lui répond.
Four digit numbers with no repeats 2010-01-02
Daniel pose la question :
I want to know how many different numbers there would be between the numbers 0000-9999 that do not have any numbers that duplicate or repeat in the number? For example the first number would have to be 0123 and the last would be 9876 (I think) Thanks!
Penny Nom lui répond.
Three digit numbers 2009-12-11
christopher pose la question :
i need a list of all 3 digit combinations with repeat of a number thnx
Penny Nom lui répond.
Two digit numbers 2009-12-10
Jim pose la question :
Can you tell me how many 2 digit numbers can be created from the following:

1118539

Penny Nom lui répond.
Three numbers that have the sum of 45 2009-12-04
amanda pose la question :
there are three numbers that have the sum of 45.The greatest of the three numbers is 2 greater than the least number.What are the numbers?what is the formula to get the answer?
Penny Nom lui répond.
6 numbers from 12 2009-11-14
david pose la question :
how many combinations of 6 numbers can there be using 12 different numbers without using the same numbers more than once?

2, 3, 5, 8, 9, 11, 12, 14, 17, 19, 20 & 31.

Penny Nom lui répond.
A sequence of letters and numbers 2009-11-10
Maria pose la question :
What is the last ten letters and numbers in the following series and how do I work it out so I can explain it to an eleven year old. J1F8M1A0M1J0J1
Robert Dawson, Claude Tardif and Harley Weston lui répond.
The cardinality of the prime numbers 2009-11-07
Justin pose la question :
Hello there, I was just wondering since the number of primes is infinite, are they equal to infinity or Aleph-null? Justin
Robert Dawson and Victoria West lui répond.
Tetrahedral numbers 2009-11-06
rena pose la question :
consider the tetrahedral numbers 1,4,10,20, etc. Can these numbers be described as a polynomial? If so, what is the degree of that polynomial?
Stephen La Rocque lui répond.
A number puzzle 2009-11-03
Jacqui pose la question :
Write a number sentence. Use every digit once, 7, 4, 3, 6, 5,10. Insert math symbols +, - , *,/ and end up with the number 3. Use parentheses if necessary.

Jacqui

Claude Tardif and Penny Nom lui répond.
The extended real numbers 2009-11-02
Justin pose la question :
Hello there, I was wondering is the set of extended real numbers a closed set or an open set?

Justin

Robert Dawson lui répond.
A number puzzle 2009-08-28
Angela pose la question :
Taking 1 through 9 and using them only once subtract them to equal 33333
Chris Fisher lui répond.
Primes 2009-06-25
Chinonyerem pose la question :
If p = 2^k - 1 is prime, show that k is an odd integer, except when k = 2. [ Hint: 3/4^n - 1 for all n >= 1.]
Robert Dawson and Penny Nom lui répond.
Is one Infinity larger than another in the extended real number system? 2009-06-24
Justin pose la question :
Hello there, I was wondering if one infinity is larger than another in the extended real number system (just like in the transfinite ordinals and cardinals with respect to infinite sets) or are all infinities the same size in the extended real number system? Thanks sooo much for answering my question! I greatly appreciate it!

All the Best,

Justin

Robert Dawson lui répond.
Triangular Numbers 2009-06-16
Chinonyerem pose la question :
Question from Chinonyerem, a student:

Each of the numbers
1 = 1, 3 = 1+2, 6 = 1+2+3, 10 = 1+2+3+4 ,...
represents the number of dots that can be arranged evenly in an equilateral
triangle:
                            .
                  .        . .
        .         .       . . .             ...
.      . .      . . .    . . . .
This led the ancient Greeks to call a number TRIANGULAR if it is the
sum of consecutive integers, beginning with 1. Prove the following facts
concerning triangular numbers:
(a) A number is triangular if and only if it is of the form n(n+1)/2 for some n >= 1
(b) The integer n is a triangular number if and only if 8n+1 is a perfect square
(c) The sum of any two consecutive triangular numbers is a perfect square
(d) If n is a triangular number, then so are 9n+1, 25n+3, and 49n+6
Penny Nom lui répond.

A number puzzle 2009-06-10
Ann pose la question :
Hey,
    I have a math problem that I can't solve which just happens to be a "must-answer". The answer to the first problem is 25,452 and the second is 21,232. I know how to add but my teacher said that there is a pattern wherein you just have to look at the set of numbers and you already know the answer without really adding it first.  Here are the condition of the problem:
   1) the first, second and fourth numbers were provided by me. ( I have no idea how this works)
   2) the third and the fifth numbers were given by the teacher. ( She's the one who knows the technique)

# She says that there is a "pattern."
 Help me please as soon as you can. Thanks!!! Here it is:
         
         5454                                   1234
   +    3636                                   5678
         6363                                    4321
         2323                                    8765
         7676                                    1234
       _______                           _________
 
 
Please answer ASAP. Thank you!!!

Penny Nom lui répond.
A number puzzle 2009-05-17
Gita pose la question :
Ok, we really need help for a homework problem due on Monday morning
I've been trying for 2 hours and can't figure it out---if I can't I'm not sure
how my son can....We've been given a puzzle with the following rules:
--The sum of each side must equal 15
--We can only use the numbers 1,2,3,4,5,6,7, 8
--A number can be used only once
(Hint: Think of all combinations of 3 numbers = 15)
--The puzzle looks like this:

_____ _______ ______

_____               ______

_____ _______ ______


Penny Nom lui répond.
15 pigs in 4 pens 2009-04-22
Melissa pose la question :
A farmer has 15 pigs. He wants to put them all into 4 pens, have an odd number of pigs in each pen, and have no pigs left over. Also, he isn't thinking about having any for dinner, as they are his pets.
How does he do it?

Robert Dawson lui répond.
Four-digit numbers 2009-04-11
jp pose la question :
How many four-digit numbers are there?
Penny Nom lui répond.
Choosing a 4 digit number at random 2009-03-24
shabkhal pose la question :
If a four-digit number is chosen at random, what is the probability that the product of the digits is 12.
Harley Weston lui répond.
wo consecutive numbers with a product of 9506 2009-02-25
sofie pose la question :
find two consecutive numbers with a product of 9506
Penny Nom lui répond.
More on cardinal numbers 2009-02-18
Justin pose la question :
Hello again, I was just wondering that since the rules of Cantor's cardinal numbers in set theory do not apply to the infinity obtained by limits in calculus (ex. x->0, y=1/x=infinity), does that mean that this infinity is the largest quantity in both calculus and mathematics?

Justin

Robert Dawson lui répond.
In what base is 3x3= 10? 2009-02-14
David pose la question :
In what base is 3x3= 10, 3x3=11, 3x3 = 12? is there a fast way to see this or do I have to create multiplication tables until I find the right one?
Penny Nom lui répond.
The page numbers in a book 2009-02-10
Nichole pose la question :
It takes 852 digits to number the pages of a book consecutively. How many pages are there?
Penny Nom lui répond.
Missing three digits in a phone number 2009-01-30
Elmer pose la question :
I have part of a phone number. the first 3 digits are 212, the last digit is four, and the three missing digits are all odd, how many numbers will I have to call to get the right number?

212- _ _ _ 4?

Stephen La Rocque and Penny Nom lui répond.
What is 301 million in word form? 2009-01-30
chyamber pose la question :
What is 301 million in word form?
Penny Nom lui répond.
How many students were there 2009-01-25
sooos pose la question :
After a math lecture in university, the students go to the cafeteria. EACH ONE of them buys a cheese sandwich and a cup of tea. They ALL pay $18.49. How many students were they?
Chris Fisher lui répond.
Multiplying negative numbers 2009-01-15
stephanie pose la question :
hi, I'm currently working in a grade 8 class where the students are learning about integers. the students were asked to find examples of how integers are applied in real life. they were able to find aplications for: adding and subtracting both positive and negative integers. however, we could not find an example of where two negative integers would be either divided or multiplied together in real life. For example: (-2)(-4)= +8. please help.
Harley Weston lui répond.
What is a group of three numbers within a larger number? 2008-12-16
kim pose la question :
My daughter brought home a question for math homework. What is a group of three numbers within a larger number?
Robert Dawson and Harley Weston lui répond.
Two numbers 2008-12-15
HERB pose la question :
I am thinking of two whole numbers. When I add them , their sum is 123. When I subtract the lesser number from the greater number their difference is 45. What are my numbers?
Penny Nom lui répond.
Three one-digit numbers 2008-12-06
JOANNA pose la question :
Could you please tell me what Adding Three One-Digit Numbers += 15 1-9 without using any number twice
Penny Nom lui répond.
Compatible numbers 2008-11-07
Erika pose la question :
When working with compatible numbers, when do you choose to round up or down? For example: In the problem: 373 divided by 4, one could round 373 to 400 and keep the 4 the same, and the answer would be 100; OR, one could round 373 down to 360 and keep the 4 the same, and the answer would be 90. When given the problem as an assignment, which way dictates how one would round, up or down?
Penny Nom lui répond.
Compatible numbers 2008-10-30
Paul pose la question :
Use compatible numbers to estimate the quotient of 23.52 and 11.04
Penny Nom lui répond.
A product of positive and negative numbers 2008-10-03
kaylou pose la question :
Suppose you were given 13 numbers and asked to find their product. Seven of the numbers were positive, and the rest were negative. Would your product be positive or negative? Why?
Penny Nom lui répond.
Compatible numbers 2008-10-02
jeff pose la question :
what exactly are compatible numbers, math problem is:
Estimate the total weight of 2 boxes that weigh 9.4 pounds and 62,6 pounds using rounding and compatible numbers. Which estimate is closer to the actual weight and why?

Harley Weston lui répond.
Compatible numbers 2008-09-25
Katie pose la question :
I am having trouble remembering how to find compatible numbers used to find the estimate in the foloowing problem. Can you please help me solve it and explain it to me? My son is in 5th grade and I need help. Thank you. Here is the problem: 2,752 / 28 estimate: 90
Penny Nom lui répond.
Two consecutive numbers 2008-09-21
tomas pose la question :
what is the two consecutive numbers with a product of 4160?
Penny Nom lui répond.
The number of digits in a really big exponent 2008-07-21
Pete pose la question :
how would you find the number of digits in a really big exponent without a calculator?
Penny Nom lui répond.
???? x ? = ???? 2008-07-20
waiyan pose la question :
????
x ?
????
using 1 2 3 4 5 6 7 8 9

Janice Cotcher lui répond.
Two numbers 2008-07-19
Jerry pose la question :
while driving in my car early this morning, i 'discovered' something and want to ask if there is an equation that would fit it...
Here it goes..... take any two numbers (with the exception of two of the same numbers), multiply the first number by 2, add the difference of the original two numbers, and the outcome will be the same as if you added the original two numbers together...
example: 3+5=8... thus, 3+3+(5-3)=8...
if you chose to reverse the numbers then..... 5+3=8... thus, 5+5-(5-3)=8

i know, simple stuff.. but i just want someone to tell me what exactly this is.. is there a 'law' that describes this?

Victoria West lui répond.
equation? 2008-07-19
Jerry pose la question :
while driving in my car early this morning, i 'discovered' something and want to ask if there is an equation that would fit it...
Here it goes..... take any two numbers (with the exception of two of the same numbers), multiply the first number by 2, add the difference of the original two numbers, and the outcome will be the same as if you added the original two numbers together...
example: 3+5=8... thus, 3+3+(5-3)=8...
if you chose to reverse the numbers then..... 5+3=8... thus, 5+5-(5-3)=8

i know, simple stuff.. but i just want someone to tell me what exactly this is.. is there a 'law' that describes this?

Victoria West lui répond.
Rational numbers 2008-07-16
hinal pose la question :
list 12 rational numbers which lie between
a) -1 and 0
b) -3 and -3

Penny Nom lui répond.
Two numbers whose sum is 25 and whose product is 120 2008-07-10
Michelle pose la question :
Solve the following problem in at least two different ways.
Find two numbers whose sum is 25 and whose product is 120.

I know how to solve this problem by creating two equations x+y=25 and xy=120 and then use substitution.

Are there any different ways of solving this problem?

Janice Cotcher and Penny Nom lui répond.
Probability and two digit numbers 2008-07-07
Peter pose la question :
What is the probability that if you multiply two randomly selected two digit whole numbers, the result is greater that 100 and less than 200. Express your answer as a common fraction.
Janice Cotcher lui répond.
Some phone numbers 2008-06-30
star pose la question :
What do you think the probability of finding the last 3 digits of a telephone number if the 1st digit begins with 7?
Janice Cotcher and Harley Weston lui répond.
Four digit combinations 2008-06-23
star pose la question :
Can you you give me a list of all 4 digit combination starting with the number 7, for example 7123,7213.
Penny Nom lui répond.
How do you change bases? 2008-06-17
Peter pose la question :
How do you change bases eg. 121 from base 3 to base 5?
Penny Nom lui répond.
Nine digit numbers 2008-05-21
Alex pose la question :
List of Nine digit numbers, that can be divided by nine?
Janice Cotcher lui répond.
A problem with numbers 2008-05-18
Peter pose la question :
When a certain number N is divided by d, the remainder is 7. If the original number N is multiplied by 5 and then divided by d, the remainder is 10, find d
Stephen La Rocque and Penny Nom lui répond.
Which is larger, 727464^512693 or 624610^518548? 2008-05-18
Yahweh pose la question :
Lets say I have two very large numbers in the form of a^b: 727464^512693 and 624610^518548

I want to calculate which number is larger, but its unreasonable to type these numbers into a spreadsheet to get an answer right away. How could I determine the larger or the two numbers efficiently?

Stephen La Rocque lui répond.
Options 1 2 4 8 16 32 64 128 etc. 2008-05-09
Abraham pose la question :
I have seen programs with options and I wish to create a similar program.

I have options 1 2 4 8 16 32 64 128 etc.
If I receive option 3 means option 1 and 2 are included. Option 20 means option 16 and 4 are included. that because of the sum of those numbers is the olny way to get the given number.

I want to know if is there a way
to know which number from options are included.
How I know:
1 is Included
2 is Included
4 is Included

Stephen La Rocque lui répond.
(1-i)ln(1+i) 2008-05-02
Kim pose la question :
I am stuck on the expansion of (1-i)ln(1+i)=(1-i)[ln(square root of 2)+i(3.14/4 = 2n3.14)]
Harley Weston lui répond.
Why can't a square be twice another square? 2008-04-20
kim pose la question :
why can't a square number double another one
Penny Nom lui répond.
The sum of some 5 digit numbers 2008-04-05
Tom pose la question :
There are 120 five-digit numbers that use all the digits 1 through 5 exactly once. What is the sum of these 120 numbers?
Penny Nom lui répond.
Compatible numbers 2008-04-04
dawn pose la question :
Need help. Estimate the quotient. Tell what compatible numbers you used.

817divided by 4 =

Penny Nom lui répond.
Operation on the mixed numbers 2008-03-26
bradley pose la question :
1) Perform the indicated operation on the mixed numbers below; write answer in simplest form:
8 1/3 – 2 1/4

2) Perform the indicated operations on the mixed numbers below; write answer in simplest form; note;: "•" denotes multiplication:
2 1/2 • 3 2/3 • 5 3

Stephen La Rocque lui répond.
The product of two numbers is 1,000,000,000 2008-03-14
Henry pose la question :
What two numbers that don't in zero when multiplied equals 1,000,000,000 and same with 1,000,000,000,000,000,000.
Penny Nom lui répond.
Some three-digit numbers 2008-03-05
Toni pose la question :
The following is a word pattern problem listed on my son's homework: write a set of three-digit numbers that contain one odd number and three even numbers. This confused the both of us because that would make his number a four digit number, right? please respond. Thanks.
Penny Nom lui répond.
One number is four less than five times another number 2008-03-05
alice pose la question :
One number is four less than, five times another number. The total of the two numbers is 50. What are the two numbers?
Penny Nom lui répond.
Fact family 2008-02-21
Anand pose la question :
Can you think of a fact family that has only two facts? Write the fact family
Penny Nom lui répond.
7, 6, 5, 4, 3 and 2 digit numbers 2008-02-10
Edward pose la question :
using the number 12345678 how many 7, 6,5,4,3 & 2 digit numbers can you make when not using any number twice in the same number
Penny Nom lui répond.
The sum of two numbers is 63 2008-02-07
kirk pose la question :
The sum of two numbers is 63. The difference between the numbers is less than 10. Find all possible pairs.
Penny Nom lui répond.
Four digit numbers containing 85 2008-02-06
Vonda pose la question :
How many four digit numbers contain the digit pattern 85 at least once and only once?
Stephen La Rocque lui répond.
A positive integer with 2007 digits 2008-02-06
Tom pose la question :
Prove that there's a unique positive integer of 2007 digits lenght, divisible by 2^2007, and made up solely by the digits 2 and 5.
Penny Nom lui répond.
Multiplying decimal numbers 2008-02-05
alwyn pose la question :
Why should when you Multiplying Decimal numbers is value becoming less and less? don't you think even decimal number is a quantity and in no chance when it multiplies its should become less or nil !!!

In fact all Multiplying and or adding the value will go up and only when you subtract and divide it should become less !!

Stephen La Rocque and Penny Nom lui répond.
Numbers and Operations 2008-02-02
Charlotte pose la question :
On the TABE (Test for Adult Basic Education) test, one of the categories is number operation. I would like to know what consist of number operation.
Penny Nom lui répond.
Choosing 10 numbers from 40 numbers 2008-01-18
Antwan pose la question :
I have 40 numbers........number 1-40. I want to know how many times i can chose 10 of those numbers without picking the same exact sequence twice if its even possible?
Stephen La Rocque lui répond.
The sum of two numbers is 36 2008-01-14
May pose la question :
The sum of two numbers is 36.
Their product is 320.
What are the two numbers?

Penny Nom lui répond.
Negative numbers 2007-12-14
mannal pose la question :
what negative integers do we use in the real world? like -30 temperature?
Penny Nom lui répond.
Improper fractions and mixed numbers 2007-12-12
Selah pose la question :
How do I change an improper fraction to a mixed number?
Penny Nom lui répond.
Imaginary roots 2007-12-09
Josh pose la question :
What is the correlation between imaginary roots (of a quadratic or other polynomial equation) and the graph of the equation? As in, how can one represent imaginary solutions graphically (and why does that work)?
Harley Weston lui répond.
Three consecutive numbers 2007-12-08
Aris pose la question :
The product of 3 consecutive numbers is 1716. Find their sum.
Stephen La Rocque lui répond.
Representing different bases 2007-12-05
Sudhir pose la question :
What does a number subscripted to another one denote? (I know that a number superscripted to another denotes the power, but the subscript number is something I have never seen before.)
Stephen La Rocque lui répond.
Compatible numbers 2007-11-19
Brendan pose la question :
Let's say you have 291/2 and its between 281/2=14 and 301/2=15. My question is where do the 14 and the 15 come from or in other words how do you get those?
Penny Nom lui répond.
x^2 + x^3 = n^2 2007-11-14
Rapin pose la question :
x and n are the whole number and less than 100 , x^2 + x^3 = n^2, please help to solve this equation.
Penny Nom lui répond.
Whole numbers and improper fractions 2007-11-13
Jennifer pose la question :
I would like to know how you convert whole numbers into improper fractions
Melanie Tyrer lui répond.
A number puzzle 2007-10-31
matthew pose la question :
please help.make 200.take 4 numbers from the numbers 1-9,place the numbers into a square containing 4 equal squares 1 number to each square.example 13
57
you now have 4 sets of numbers 13 ,57 ,reading across and 15 , 37. reading down i have to find 2 or more ways of making i have to find the numbers that when added together using this method make 200.you can only use each number once.

Penny Nom lui répond.
I am a prime number 2007-10-31
sherry pose la question :
I am a prime number. I can not be shared fairly except by one group. My array can only be one row. array can only be one row. Can you please explain this out to me so I can help my child with it. Thanks
Penny Nom lui répond.
Compatible numbers 2007-10-30
Gary pose la question :
How do I estimate the quotient. and then tell what compatible numbers were used. 703/7
Penny Nom lui répond.
Complex numbers 2007-10-27
Dylan pose la question :
My problem is to prove:

|z|^2 = zz* Where z is the complex number x + iy and z* is it's complex conjugate x - iy.

If the absolute value of i is 1, then it looks like: |z|^2 = |x+y| |x+y| = x^2 + 2xy + y^2

And zz* = x^2 + y^2. for these to be equal, 2xy = 0. This seems wrong to me. What am I doing wrong?

Penny Nom lui répond.
The largest number you can add ... 2007-10-26
Dryden pose la question :
What is meant by asking what is the greatest number you can add to that number without having to regroup in any place?
Penny Nom lui répond.
4 odd numbers that equal 21 2007-10-15
gary pose la question :
4 odd numbers that equal 21
Penny Nom lui répond.
Equivalent mixed numbers 2007-10-02
negra pose la question :
an average-sized person can burn about 6 1/2 calories a minute while ridinng a bike. Which of the following is equivalent to that amount?
a) 1 2/2

b) 5 6/2

c) 6 2/4

d)6 2/6

Stephen La Rocque and Penny Nom lui répond.
How many ten thousands makes one million? 2007-10-02
Payton pose la question :
how many ten thousands makes one million?
Penny Nom lui répond.
Four digit odd numbers 2007-09-26
sonia pose la question :
how many 4 digit odd numbers can be made using the digits 2, 5, 7, and 8
Penny Nom lui répond.
A fraction that cannot be simplified 2007-09-17
Kevin pose la question :
Make a fraction that cannot be simplified and has 24 as its denominator?
Stephen La Rocque and Chris Langdon lui répond.
How many two digit numbers contain at least one 7? 2007-09-06
Janet pose la question :
How many two digit numbers contain at least one number seven?
Penny Nom lui répond.
Compatible numbers 2007-09-05
Beth pose la question :
Estimate using compatible numbers:

2 X 3978 =

102/25 =

Stephen la Rocque lui répond.
Compatible numbers 2007-09-01
andrea pose la question :
Estimate using compatible numbers. 249 /64
Harley Weston lui répond.
How many three button codes are possible? 2007-08-23
kim pose la question :
A home security device has ten buttons and is disarmed by pushing three buttons in sequence. How many three button codes are possible if each button can be pushed more than once in the code?
Stephen la Rocque lui répond.
Four digit numbers 2007-07-30
Spence pose la question :
My son has a problem that he has been unable to figure out. I was asked to try, but I am confused on how to get the last two parts. Any help would be great.
Thanks
Spence

How many four-digit numbers can be formed under each condition?

A) The leading digit cannot be zero: 9(10)(10)(10)=9000

B) The leading digit cannot be zero and no repetition of digits allowed: 9(9)(8)(7)=4536

C) The leading difit cannot be zero and the number must be less than 5000: ??

D) The leading digit cannot be zero and the number must be even: ??

Penny Nom lui répond.
Multiplcation of two negative numbers 2007-07-26
Brett pose la question :
Someone asked a question about multiplication and division of two negative numbers yielding a positive result here: http://mathcentral.uregina.ca/qq/database/QQ.09.99/butler1.html I was not fully happy with the explanation b/c I want to give me daughter a real-world example and I can't seem to find one.

The following illustrates why multiplying negative numbers has become difficult to explain:

2 X 2 = 4

----(-4)---(-2)---0---2---4
In this example we start with 2 and then want 2 more of them. When we move across the number line from 2 to our answer, which is four, we have moved only 2 units to the right.

-2 X -2 = 4

----(-4)---(-2)---0---2---4
In this example we start with -2 and then want -2 more of them. When we move across the number line from -2 to our answer, which is four, we have moved 6 units to the right.

How can the phenomenon of multiplying two negative numbers being more powerful than multiplying two positive numbers be explained? -Brett

Stephen La Rocque and Harley Weston lui répond.
(1 - i)^5 2007-07-24
sofia pose la question :
Compute the given arithmetic expression and give the answer in the form a + bi where a,b element in R. 1. (1 - i)^5
Harley Weston lui répond.
A complex number in polar form 2007-07-23
roland pose la question :
write the given complex number z in polar form lzl(p+qi) where lp + qil=1 for 3 - 4i.
Harley Weston lui répond.
If A is 1 and Z is 26 2007-06-29
Sue pose la question :
If A is 1 and Z is 26 is there a word that when you multiply all the numbers instead of letters adds up to a million?
Chris Fisher and Harley Weston lui répond.
The number of blocks on the face of a pyramid 2007-06-27
Heather pose la question :
I was wondering if there was an equation in order to calculate the number of blocks on the face of a pyramid. For example:

X
X X
X X X

Is there a specific formula in order to calculate that there are 6 blocks here? Thank You, Heather

Harley Weston lui répond.
Exponential form: x^y 2007-06-22
Kishor pose la question :
whats the easy way to calculate X raised to Y where y is much greater than x.
Stephen La Rocque lui répond.
Simplifying complex denominators 2007-06-21
Krys pose la question :
How do I simplify completely? ((4+i ) / (3+i )) - ((2-i ) / (5-i ))
Stephen La Rocque lui répond.
Compatible numbers 2007-06-15
meryl pose la question :
what is a compatible number? ex.6,321/8
Penny Nom lui répond.
Repeating decimals and rational numbers 2007-05-31
lil pose la question :
Why are repeating decimals considered rational numbers?
Penny Nom and Gabriel Potter lui répond.
What is the highest known numerical value? 2007-05-29
Peg pose la question :
What is the highest known numerical value?
Gabriel Potter lui répond.
Positive rational numbers 2007-05-22
Rebecca pose la question :
What is the definition of positive rationals/positive rational numbers?
Stephen La Rocque lui répond.
Fractions and mixed numbers 2007-05-08
Amanda pose la question :
I am having truble with this problem: WRITE THE FRACTION 8/3 AS A MIXED NUMBER
Stephen La Rocque lui répond.
The sum of two or more consecutive integers 2007-04-26
nana pose la question :
some numbers can be written as the sum of two or more consecutive integers(we consider positive integers only) other number (eg. 4) cannot be express in this way.
let us call a number which can be expressed in this way a SOALTCI( sum of at least two consecutive integers)
a). list the first few SOALTCI and conjecture the general formula.
b). Prove that any number of the given form(in your answer (a)) is a SOALTCI

Penny Nom lui répond.
24 four-digit numbers 2007-04-21
Megon pose la question :
Of the 24 four-digit numbers formed from the digits 1,2,3,&4 is there an easier way to find their sum other than writing them all out and totaling them by hand?
Stephen La Rocque lui répond.
Three digit numbers 2007-04-19
katelyn pose la question :
How many 3 digit numbers can be created if each number is a multiple of 10?
Penny Nom lui répond.
How many different ways can she purchase candy? 2007-04-18
fathia pose la question :
Stephanie can buy lemon candies for 3 cents each and chocolate candies for 5 cents each. Stephanie has 64 cents to spend. How many different ways can she purchase candy if she wants to spend all her money?
Stephen La Rocque lui répond.
-12/(7 - i) 2007-04-18
Diana pose la question :
Perform the operation. Write all answers in a + bi form.

-12
---------
7 - radical -1

Penny Nom lui répond.
A subtraction problem with negative numbers 2007-04-12
Bridget pose la question :
my daughter has to change a subtraction problem with negative numbers in it to an addition problem with negative numbers and then solve the problem such as: -4-(-2)=?
Penny Nom lui répond.
The sum of two rational numbers 2007-04-04
Fathia pose la question :
Hi, Could please help me with this question? Prove that if a and b are rational numbers then a + b is a rational number. Thank you,
Stephen La Rocque lui répond.
Two consecutive odd numbers 2007-03-19
Alicia pose la question :
The sum of two consecutive odd numbers is divisible by 4.

Can you make similar statements about the sum of three consecutive odd numbers and of four consecutive odd numbers? Generalize your findings.

I tried :
n= smallest odd number therefore the next = n, n+2 and the second =n+4 and the third = n+6 Now n+ n+2+n+4+n+6/4
Please help I am not sure of this I am just trying

Penny Nom lui répond.
Using complex numbers 2007-03-12
Kara pose la question :
Do you use complex numbers in your job?
Stephen La Rocque and Penny Nom lui répond.
Seven digit account numbers 2007-03-09
Miranda pose la question :
Account numbers for the Central Oil Company consist of seven digits. If the first digit cannot be 0, how many account numbers are possible? Thank you so much!!!
Penny Nom lui répond.
Four digit numbers 2007-02-26
Anton pose la question :
Hello! I would like to ask you how many combinations are possible using 4 digit number. # # # # (1...9).There is no zero,numbers don't repeat,second and forth are odd.Thanks
Penny Nom lui répond.
What are the two numbers? 2007-02-25
Thomas pose la question :
Two numbers have a total of 32 and a difference of 14. What are the two numbers?
Stephen La Rocque lui répond.
Exponential form of complex numbers 2007-02-12
Austin pose la question :
When dealing with imaginary numbers in engineering, I am having trouble getting things into the exponential form. The equation is -1+i now I do know that re^(theta)i = r*cos(theta) + r*i*sin(theta). Just not quite understanding the order of operations. Thanks
Penny Nom lui répond.
Prime numbers 2007-01-16
A student pose la question :
are the product of two prime numbers always prime?
Penny Nom lui répond.
21 pigs and only 4 pig pens 2007-01-11
Emma pose la question :
i was given this pig pen problem. if there are 21 pigs and only 4 pig pens how can you distribute the pigs into the pens and have an odd number of pigs in each pen. it doesn't seem possible to me. i need help!
Haley Ess lui répond.
A word problem 2007-01-05
Harold pose la question :
the sum of one number plus four less than five times another is 26.
Penny Nom lui répond.
Phone number combinations 2007-01-04
Efrain pose la question :
i want to know the different combinations for numbers 7 digits the number are 0-9 its basically the different phone number combinations
example: 555-6689 or 565-4896 i just want to know for many different combinations are there....

Penny Nom lui répond.
A word problem 2006-12-30
Harold pose la question :
one number is less than 5 times another number. sum of the numers is 26
Penny Nom lui répond.
1, 3, 6, 10... 2006-12-15
Lisa pose la question :
What is the nth term for the sequence, '1, 3, 6, 10...' and could you say how you got the anwser.
Stephen La Rocque and Penny Nom lui répond.
The absolute value of imaginary and complex numbers 2006-12-11
Keith pose la question :
i don't get how to find the absolute value of imaginary and complex numbers here is an examples from the text book the answers are given but they don't show the work so i can follow along just show me the work please and explain how it is done

problem
3+4i

Stephen La Rocque and Penny Nom lui répond.
Four two-digit numbers 2006-12-02
Mary pose la question :
I have four two-digit numbers written on my paper. The sum of these 4 numbers is less than 100.

True or False - each number is less than 25

If all the numbers on my paper are different, what is the largest (two-digit) number I could possibly have written.

Stephen La Rocque lui répond.
The sum of prime numbers 2006-11-28
Rambabu pose la question :
I would like to know the sum of prime numbers below some X say 10000. Provide me with easy way to get this done
Penny Nom lui répond.
Number combinations 2006-11-09
Bob pose la question :
how can I find out all the different combinations for the numbers 8168 example 8168, 8868, 1688,1886,6818 etc. how many are their.
Penny Nom lui répond.
The real numbers with decimal representations consisting of all 1s. 2006-10-29
Ivessa pose la question :
Determine if the following set is countable or uncountable : the real numbers with decimal representations consisting of all 1s.
Steve La Rocque and Walter Whiteley lui répond.
Four digit combinations 2006-10-26
Sasha pose la question :
CAN YOU LIST ALL OF THE POSSIBLE 4 DIGIT COMBINATION'S, THAT DO NOT START WITH 0?
Stephen La Rocque lui répond.
Three consecutive odd numbers 2006-10-21
Pige pose la question :
The sum of three consecutive odd numbers is 387. Find the numbers.
Penny Nom lui répond.
Write the digits 1 to 9 on index cards, ... 2006-10-19
Kim pose la question :
Write the digits 1 to 9 on index cards, arrange the cards to form three 3-digit numbers (using each card only ONCE). The third number is the sum of the first two numbers ** how many different ways can you do this?? **
Penny Nom lui répond.
The sum of two whole numbers is 72 2006-10-18
Kelin pose la question :
The sum of two whole numbers is 72. Their differences is 48. What are the two numbers.
Stephen La Rocque and Penny Nom lui répond.
How many students are in the class? 2006-10-12
Lillian pose la question :
There is one more boy than girls in the class, how many students in all?
Stephen La Rocque lui répond.
Four fours 2006-10-08
Prabh pose la question :
Find out 10 BEDMAS problems using order of operation with only four 4's in the problem. The solution must be the digits 1-10. Example 44/44=1
Stephen La Rocque lui répond.
What number could it be? 2006-09-27
Jonathan pose la question :
What number could it be?

It is between 12000 and 13000
Its hundreds, ten and ones digits are all odd
the sum of all its digits is 12.

Steve La Rocque and Claude Tardif lui répond.
A remainder of 1 2006-09-13
Juan pose la question :
What is the smallest counting number which when divided by any of the numbers 2,3,4,5,6,,7,8,9 and 10, leaves a remainder of 1?
Paul Betts and Claude Tardif lui répond.
How many different three-digit numbers can you make using ... 2006-09-12
Beth pose la question :
How many different three-digit numbers can you make using the numbers 2, 5, 7, 8 using the digits any number of times?
Stephen La Rocque and Penny Nom lui répond.
How many thousands are in ten million. 2006-08-24
Echoe pose la question :
How many thousands are in ten million.
Stephen La Rocque lui répond.
The last digit of a six-digit number is 2. 2006-07-09
GG pose la question :
The last digit of a six-digit number is 2. If the 2 is moved to the start of the number, the new six-digit number is only a third of the original number. Find the original number.
Paul Betts, Chris Fisher, Penny Nom lui répond.
As close to 841 as possible 2006-06-21
Alan pose la question :
using these numbers by either add subtract division multiply come up with the answer as close to 841
100 25 10 7 6 3

Paul Betts lui répond.
express the number 100 using the same figure 6x 2006-06-17
A student pose la question :
express the number 100 using the same figure 6x
Stephen La Rocque lui répond.
how do i find i^22? 2006-06-12
Sky pose la question :
how do i find i22?
Sky

Stephen La Rocque lui répond.
What is the sum of the first 100 whole numbers? 2006-05-31
Jo pose la question :
what is the sum of the first 100 whole numbers?
Natasha Glydon, Paul Betts and Penny Nom lui répond.
Three questions 2006-05-17
Diane pose la question :

Question:

I

What is the largest real number less than 10?
1- there is no largest real number less than 10 or
2- 9
or
3- 9.999...

II

Given that not (a and b) implies c.What does
(not c) imply ?
1-a and b
2- (not a and b) or (a and not b)
3- (not a and b) or (a and not b)
4- a or b

III

What are the x and y intercepts of the equation
2x + 3y -6 =0 ?
1- x=3 and y=2
2- x=6 and y=-2
3- -3x + 2y =0


Paul Betts lui répond.
What is the radius of the sphere? 2006-05-17
Jimmy pose la question :
The surface area and volume of a sphere are both equal to the product of 4-digit whole numbers and pi. What is the radius of the sphere?
Stephen La Rocque lui répond.
Four digit numbers 2006-03-25
Kenvin pose la question :
i was wondering how many possibilities is there for a four digit number with the same first two numbers and the last two is not one of the first with numbers 1-9
Stephen La Rocque lui répond.
Five digit numbers 2006-03-25
Tony pose la question :
Can someone tell me how many unique five digit numbers can be generated using 0-9?
Stephen La Rocque and Penny Nom lui répond.
given that p is a prime and p|a^n, prove that p^n|a^n 2006-03-24
Janna pose la question :
given that p is a prime and p|an, prove that pn|an
Stephen La Rocque lui répond.
A nine digit number 2006-03-06
Ryan pose la question :
What is the total number of possible combinations of a nine digit number (ie., social security number) including repeating numbers?
Stephen La Rocque and Penny Nom lui répond.
The meaning of numbers 2006-03-03
John pose la question :
I'm having a philosophical debate on the meaning of numbers, equations, or the world of math in general. Would it be possible if you could help me by giving me a resource that talks about it or if you have your own opinion I would be most grateful.
Harley weston lui répond.
A triangle of 50p pieces 2006-02-22
Stuart pose la question :

Ok, so i am collecting 50p pieces and arranging them on my desk in the shape of a triangle.
eg
50p
50p 50p
50p 50p 50p
50p 50p 50p 50p

I want to work out how much money I'm saving just by knowing how many rows of coins there are. If i can work out how many coins there are just by knowing how many rows I have I can just divide by 2 to find out the amount in dollars.


Penny Nom lui répond.
sinh(i/2) 2006-02-09
Louis pose la question :
How can you set up an equation to find sinh(i/2)
Penny Nom lui répond.
Write the array for 3 2006-01-29
Jamie pose la question :
My son is a 5th grader and came home with a math question that I can't answer. The question stated - "Write the array for 3" I don't understand how this can be done with just one number.
Penny Nom lui répond.
GCF, LCM, primes and the ladder method 2006-01-07
Linda pose la question :
How would I teach both finding the GCF and LCM with prime factors...I recall the ladder method vaguely.???
Penny Nom lui répond.
The square root of i 2005-11-30
Kevin pose la question :

If the square root of -1 is i, what is the square root of i?

How can you find the log of a negative number?

What is the log of -1?


Claude Tardif lui répond.
100, 000 or 100 000 2005-11-21
Jacqui pose la question :
I am a parent of a grade 6 student and need to confirm that commas are no longer used when writing numbers (the Canadian method). For example, how do you write one hundred thousand? 100, 000 or 100 000
Diane Hanson and Harley Weston lui répond.
5 digit numbers that include 5 and exclude 8 2005-11-04
Paul pose la question :
figure out how many 5 digit numbers are there that include the digit 5 and exclude the digit 8. show me how to do get to the answer. (23816)
Paul Betts lui répond.
A sum of 75 and a difference of 13 2005-10-30
Vicki pose la question :
Find two numbers that have a difference of 13 AND A SUM OF 75? We need to learn about the formula or procedure.
Penny Nom lui répond.
Two 3-digit numbers 2005-10-27
Melissa pose la question :
I can't figure this out, The question is

Find the greatest possible difference of two 3-digit numbers that use the digits 3, 5, 7 and 9 only once in each number.

The teacher says the answer is 62 but I can't figure out how she did it. Can you help.

Claude Targif and Penny Nom lui répond.
What number am I? 2005-10-27
Samantha pose la question :
First 8 is added to me, then I am multiplied by 6. Then 40 is subtracted from me. Finally, I am divided by 10. The result is 11. What number am I?
Penny Nom lui répond.
Improper fractions 2005-10-21
Joe pose la question :
Is a whole number an improper fraction. For example, is 4 an improper fraction? Is 4 an improper fraction? Is 4 the simplest form for 20/5? Are 4 and 20/5 equivalent fractions?
Penny Nom lui répond.
Divisibility by each of the first ten counting numbers 2005-10-17
Simon pose la question :
determine smallest positive integer that is divisible by each of the first ten counting numbers
Penny Nom lui répond.
Whole numbers 2005-10-10
Kirk pose la question :
Can the sum of two or more whole numbers be less than any of those numbers?
Penny Nom lui répond.
Four digit numbers 2005-10-09
David pose la question :
I had a discussion with my father about how many four digit numbers you can make with 0-9. He was saying that there is 12,000 and that seems quite high for me. But can you tell me how many there are, and what exactly are they?
Penny Nom lui répond.
Three digit numbers 2005-10-04
Jordan pose la question :
i need to know how many 3 digit numbers there are
Penny Nom lui répond.
Use the numbers and symbols only once to come up with the answer 2005-09-19
Kaitlyn pose la question :

Question: I have math problem that no one in my class was able to solve but the teacher said it does have an answer. We need to use the numbers and symbols only once to come up with the answer:

36 18 12 6 + - ( )

______________________________ = 41


Claude Tardif lui répond.
What is my number? 2005-09-16
Bob pose la question :
I have 6 digits. 100,000th digit is one less than my ones digit, eight greater than my 1,000's digit, twice my tens digit and four times my 100s digit. My 10,000s digit is 0 What is my number?
Penny Nom lui répond.
A two digit prime 2005-09-12
A student pose la question :
The units digit of a two-digit prime number is not greater than the tens digit by 3.If the positions of the two digits are interchanged, then the new number formed is greater than the original number by more than 18. Find the value of the original prime number.
Penny Nom lui répond.
Adding positive and negative numbers 2005-09-03
Billy pose la question :
46 + -24 =
-38 + -22 =
19 - 20 =

Penny Nom lui répond.
1,4,9,1,6,2,5,3,6,4,9,6,4,8,1 2005-08-30
Liz pose la question :
Find the next four numbers to the sequence 1,4,9,1,6,2,5,3,6,4,9,6,4,8,1,___,___,___,___.
Penny Nom lui répond.
1_2_3_4_5_6_7_8_9 = 1: Fill in the blanks 2005-08-26
James pose la question :
Try to replace the blanks below with + or - to make the statement correct
1_2_3_4_5_6_7_8_9 = 1

Penny Nom lui répond.
Their sum equals their product 2005-08-21
Claudio pose la question :
Find a thousand natural numbers such that their sum equals their product.
Penny Nom lui répond.
Can one place 15 integers around a circle... 2005-08-20
Hernan pose la question :
Can one place 15 integers around a circle so that the sum of every 4 consecutive numbers is equal either to 1 or 3?
Claude Tardif lui répond.
Which number is greater? 2005-08-18
Dante pose la question :
Which number is greater: 888....88 X 333...33 or 444.......44 x 666........67 (each of the numbers has 1989 digits)?
Claude Tardif lui répond.
Converting rational numbers 2005-08-07
Joe pose la question :

I am helping my son with his converting rational
numbers in the form of A/B where A and B are integers and
B not equal to zero

I think I am doing this right but I am not sure so
below are the problems and our answers are beside
them, please let me know if these answerers are correct


Penny Nom lui répond.
A set of numbers whose sum is 1 2005-07-21
Santos pose la question :
Does there exist a set of numbers whose sum is 1, and the sum of whose squares is less than 0,01?
Paul Betts lui répond.
Seven digit numbers 2005-06-23
Ky pose la question :
How many number combinations are possible for a 7-digit number? How many people in a population can exist without having to exceed the seven digit combinations and having to revert to a 8 or 9 digit number?
Penny Nom lui répond.
Primes 2005-06-04
Fede pose la question :
Prove that there is not a number prime p, for which the numbers p+5 and p+10 are prime too.
Penny Nom lui répond.
The numbers p and 8p^2 +1 are prime. 2005-05-30
Antonio pose la question :
The numbers p and 8p2 +1 are prime. Prove that the number 8p2+2p+1 is also a prime number.
Claude Tardif and Penny Nom lui répond.
5+5+5=550 2005-05-30
Bill pose la question :

5+5+5=550

This is a math equation that is not true. How can it be made to be true by using only one line?
Leeanne Boehm lui répond.

Which is larger, 31^11 or 17^14? 2005-05-29
Linda pose la question :
Which number is greater 3111 or 1714 (without using calculator).
Chris Fisher and Paul Betts lui répond.
50^99 and 99! 2005-05-23
Romel pose la question :
Which number is greater, 5099 or 99!
Claude Tardif lui répond.
Which is smaller 2^175 or 5^75? 2005-05-18
Sarah pose la question :
Which is smaller 2175 or 575? without using a calculator.
Claude Tardif lui répond.
A pattern of numbers 2005-04-21
Claire pose la question :
We need to be able to discover the nth term in a pattern of numbers and explain how we did it. We have arrived at the formula 1+2+3+...n=n(n+1)/2 for trios where a trio series is;

3 has the following trio (1,1,1)
4 has the following trios (2,1,1), (1,2,1), (1,1,2)
5 has the following trios (3,1,1), (1,3,1) (1,1,3) (1,2,2) (2,2,1) (2,1,2)

So the series is as follows

Penny Nom lui répond.
Some four digit numbers 2005-04-16
Lori pose la question :
can u give me every possible combination of four numbers starting with the number three
Penny Nom lui répond.
Prime numbers 2005-04-11
Jesule pose la question :
Given that p, p+10 and p+14 are prime numbers, find p.
Claude Tardif and Chris Fisher lui répond.
Rational functions 2005-04-05
Nicole pose la question :
My name is Nicole and I am a teacher at Weyburn Comprehensive School. I am currently teaching both Math B30 and Calculus 30 at the school and I have a question about rational functions. I know that if a rational function (by definition) has common factors in the numerator and the denominator then it is not a rational function (math b30) however in calculus this common factor creates a hole in our graph. Can you explain to me why a common factor or constant does not give us a rational function?
Penny Nom and Leeanne Boehm lui répond.
When dividing a 3-digit number by a 1-digit number... 2005-01-22
A student pose la question :
When dividing a 3-digit number by a 1-digit number, for what divisors can you get a remainder of 8?
Penny Nom lui répond.
Numbers that John likes 2005-01-16
Garrett pose la question :
John likes 400 but not 300; he likes 100 but not 99; he likes 3600 but not 3700. Which does he like?

900
1000
1100
1200

Penny Nom lui répond.
Six digit numbers with at least one 7 2004-12-27
Behzad pose la question :
How many six-digit numbers contain at least one 7 in their decimal expansion?
Penny Nom lui répond.
Making the number 99999 2004-12-22
Lisa pose la question :
Make as many equations as possible to make the number 99999 using all of the numbers 0-9 but only once per equation. example 01234 + 98765 = 99999 she needs to make 150+ equations.
Paul Betts lui répond.
Some numbers less than 100 2004-09-17
Paul pose la question :
for how many numbers less than 100 is the digit in the ones place half the digit in the tens place?
Penny Nom lui répond.
Half way between 2004-09-08
Ben pose la question :
Find the number halfway between the number shown

751,843

Penny Nom lui répond.
A big number 2004-08-26
Mark pose la question :
If you have $999,999,999,999,999,999,999,999,999,999.99 and you find a pennie how much is that and can you give me the name of it.
Penny Nom lui répond.
A big number 2004-08-08
Carter pose la question :
Can you tell me what number this is?
10,000,000,000,000,000,000,000,000,000,000,000,000

Where can I find the names for big numbers?

Penny Nom lui répond.
Four digits 2004-07-10
Rob pose la question :
I am always interested to know how the for digit works. I mean, you have correctly put it that the total numbers obtain from 0000 thru 9999 are 10,000. My question is, is there any simple formula to turn this simple 4 figures of say: 1234.
Penny Nom lui répond.
Factoring integers 2004-07-02
A student pose la question :
After looking at all the info I could get about NFS, I still have some questions that are unsolved:

First of all: If someone found an algorithm that has a worst case running time of N*Log(N) to factor an integer n into his divisors, would it be quicker or slower then the number field sieve algorithm?

secondly, what exactly is the time complexity of the Number Field Sieve algorithm, if I would factor an integer n?

Claude Tardif lui répond.
Aunt Lucy's letter 2004-06-27
Olivia pose la question :
"Dear bob, now that i am getting on (70 today). i want to give you some of my money, i shall give you a sum each year, starting now. you can choose which of the following schemes you would like me to use.

a) £100 now, £90 next year,£80 the yaer after and so on.
b) £10 now, £20 next year, £ 30 the next year and so on.
c) £10 now, 1.5 times as much next year, 1.5 times as much the year after that and so on.
d)£1 now, £2 next year, £ 4 the year after, £8 the year after and so on.

of course these schemes will only operate while i am alive. i look forward to hearing which scheme you choose and why! Best wishes, Aunt Lucy."

Penny Nom and Claude Tardif lui répond.
Divisors of 2n+1 and 3n+9 2004-06-02
Chris pose la question :
Amy selected a positive integer N and wrote numbers 2xN+1 and 3xN+9 on a whiteboard. Boris looked at the two numbers on the whiteboard and wrote in his notebook a positive integer that divides each of them. Find all possible numbers that Boris could have written in his notebook.
Chris Fisher and Penny Nom lui répond.
Numbers around a circle 2004-03-28
Rebecca pose la question :
my maths question is use the numbers 1,2,3,4,5,6 and 7 place each number in a circle so each line adds up to 12. There are seven circles, six on the outside and one in the middle. Each number lines up with the middle number and the outside numbers line up with the one directly across from it as if a line was going through the middle number circle.
Penny Nom lui répond.
z^2 = 3 - 4i 2004-03-26
John pose la question :
Solve: Z^2 = 3 - 4i
Harley Weston lui répond.
Large exponents 2004-03-26
John pose la question :
I have encountered problem with the lack of memory every calculator seems to have. No calculator, on or off the computer, I've found has the amount of memory or writing space to calculate the sums I want to solve. The sums are in great importance for my continued progress. The sums are following:

16777216^1310270, 16777215^995328, 16777215^786432 and 16777215^480000

Claude Tardif lui répond.
Some number problems 2003-12-24
Shruthi pose la question :
1. Write 31 using 3 five times
2. Write 100 using all the 10 digits just once
.
.
.

Penny Nom lui répond.
400, 100 and 2500 2003-12-21
A student pose la question :
A person likes 400 but dislikes 300
He also likes l00 but dislikes 99
He also likes 2500 but dislikes 2400

Which of the following does John like
900, 1000, 1100 or 2400


Penny Nom lui répond.
Four digit numbers 2003-11-20
Rob pose la question :
Hello, I sure hope you can answer this question. I got into a discussion at work about how many possible combinations there are for a four digit number using the numbers 0 through 9. This was in reference to a discussion about the lottery. I said there was only 10,000 possible combinations. My friend said that there was many times more, but he could not remember how to figure it out. The answer and the formula would be greatly appreciated.
Penny Nom lui répond.
The product of the page numbers 2003-11-07
Morgan and Mom pose la question :
You open a book and the product of the page numbers is 12,656. What are your page numbers?
Penny Nom lui répond.
Pairs of prime numbers 2003-10-13
Nikolas pose la question :
Use pairs of prime numbers to find all the numbers less than 50 that have only two prime factors. Make an organized list.
Penny Nom lui répond.
Primes 2003-09-23
Julie pose la question :

#1. Suppose you are given a number (n) and told that 1 and the number (n) divide into (n). Does that mean (n) is prime?

#2. Suppose that p is a prime number greater than or equal to 3. Show that p+1 cannot be a prime number.


Penny Nom lui répond.
The Sieve of Eratosthenes 2003-09-18
Lynn pose la question :
My daughter has been asked to find all the prime numbers by using the Sieve of Eratosthenes. I have no understanding what this means.
Penny Nom and Claude Tardif lui répond.
How high does the sequence of numbers go 2003-09-07
Shayna pose la question :
My quetion is "How high to the sequence of numbers go"?
e.g. one, ten, hundred, thousand, million, billion, trillion, and from there is my queston.

Penny Nom lui répond.
Two row arrays 2003-09-03
Madalyn pose la question :
My question is, what are #'s that can not be arranged in two row arrays called?
Penny Nom lui répond.
X.9999... and X+1 2003-08-23
David pose la question :
I have read your answers to the questions on rational numbers, esp. 6.9999... = ? and still have a question: The simple algebraic stunt of converting repeating decimals to rational numbers seems to work for all numbers except X.999999.... where X is any integer. The fact that the method yields the integer X+1 in each case seems to violate the completeness axiom of the real numbers, namely that there is no space on the number line which does not have an number and conversely that every geometric point on the number line is associated with a unique real number. In the case of 3.999... for example, it seems that both the number 4 and the number 3.9999.... occupy the same point on the number line. How is this possible???
Penny Nom lui répond.
39 consecutive natural numbers 2003-08-19
A student pose la question :
Prove that among any 39 consecutive natural numbers it is always possible to find one whose sum of digits is divisible by 11.
Penny Nom lui répond.
nine digit numbers 2003-07-23
John pose la question :
With a nine digit number, with each of the nine digits having a possibility of 10 different numbers then what is the total number of possible mathematical variations in the nine digit number. i.e. Social Security numbers have nine digits and if each of the nine digits have a possibility of being any one of ten numbers, i.e. 0,1,2,3,4,5,6,7,8,9. Then what is the formula to calculate the maximum possible number of variations in this nine digit number and what is the mathematical maximum possible number of variations of this nine digit number?
Penny Nom lui répond.
Numbers John likes 2003-06-20
Steve pose la question :
John likes 400 but not 300; he likes 100 but not 99; he likes 2500 but not 2400.

Which does he like?
900
1000
1100
1200

Penny Nom lui répond.
The sum of two numbers is 5 and their difference is 2003-06-16
Akhil pose la question :
The sum of two numbers is 5 and their difference is 11. What is the product of the two numbers?
Penny Nom lui répond.
Real numbers 2003-05-09
Sirena pose la question :
what is a "real" number
Penny Nom lui répond.
Harmonic numbers 2003-03-19
Becky pose la question :
Harmonic numbers are Hn = 1 + ? + 1/3 + . . . + 1/n

Use induction to prove the following theorem:
For all natural numbers n, H1 + H2 + . . . + Hn = (1+n)Hn - n

Penny Nom lui répond.
1 followed by a million zeros 2003-03-19
David pose la question :
What do you call the number represented by the numeral '1' followed by one million zeros?
Penny Nom lui répond.
1575 2003-03-13
Sharyn pose la question :
find 6 integers that when multiplied with each other equal +1575 and when the same integers are added together equal zero. there are more positives than negatives and one of the numbers is a double digit between 10 and 20
Penny Nom lui répond.
Harmonic numbers 2003-03-12
Becky pose la question :
What can you tell me about the limit of harmonic numbers as it reaches infinity?
Penny Nom lui répond.
Hundreds, thousands, millions,... 2003-02-19
Karissa and Jasmeen pose la question :
hundreds, thousands, millions, - can you help us with the rest of this sequence - we are trying to find the largest number
Penny Nom lui répond.
6 digit numbers from 0,0,2,2,4,4 2003-01-23
Amanda pose la question :
How many 6 digit numbers can you make from the numbers 0,0,2,2,4,4, giving that 0 cannot come first. The number has to contain 2 twos, 2 fours, and 2 zeros.
Penny Nom and Claude Tardif lui répond.
What is larger than infinity? 2003-01-12
Dana pose la question :
What is larger than infinity?
Claude Tardif and Harley Weston lui répond.
ALL of the possible 3 digit combinations of 0 - 9 2002-12-09
Melissa pose la question :
I need to find out ALL of the possible 3 digit combinations of 0 - 9. Eg: 000,001,002,003. This is going to take me VERY LONG time. Any suggestions.
Penny Nom lui répond.
Making 7 2002-11-26
Bill pose la question :
At one time I had the answer to this math equation but I have lost it and can't seem to regain it. I may be going at it wrong but as I recall there were the numbers 1 through 4, and the std operators, +, -, /, *, and parentheses. The object was to make the four numbers with the operators equal to 7. Each number and operator may be used only once. My 9 year old loves these quizzes but I can't give it to her if I can't provide the correct answer.
Claude Tardif lui répond.
4 x abcd = dcba 2002-11-20
A student pose la question :
Say you have a four digit number (e.g. abcd) and you multiply the number by 4. The answer you get will be the reverse order of the number you started with (dcba). What is the number? The four digits (a,b,c,d) cannot be the same number or cannot be repeated.
Penny Nom and Claude Tardif lui répond.
Why is 5 divided by 1/7 greater than 5 divided by 2/3? 2002-11-19
Elizabeth pose la question :
  1. Without performing the division, explain why 5 divided by 1/7 is a number greater than 5 divided by 2/3.
  2. Is the least common multiple of two prime numbers always their product? Why or why not?

Diane Hanson and Penny Nom lui répond.
The product of two consecutive numbers is 41 more than their sum 2002-11-15
A mother pose la question :
THE PRODUCT OF TWO CONSECUTIVE NUMBERS IS 41 MORE THAN THE SUM OF THE NUMBERS. WHAT ARE THE NUMBERS?
Penny Nom lui répond.
Prime numbers 2002-11-08
Lonna pose la question :
Is 2 a prime number or not??
Why is 1 not considered a prime number?

Penny Nom lui répond.
7+8+9+...+1000 2002-09-11
Shirley pose la question :
My question is what is the formula for adding up numbers when you don't start with number 1? For example 3 + 4 + 5 + 6 = 18. But how could you arrive at the answer without adding all the numbers?
Penny Nom lui répond.
Subsets of the Real Number System 2002-09-07
Christine pose la question :
In looking for a French equivalent of the word "integer", I found that the word seems not to actually exist in French, and that Canadian schools use the term "natural number" to describe what we have been trained to call "whole numbers," while using the term "whole number" to describe what we have been trained to call "integers."
Claude Tardif lui répond.
Social security numbers 2002-09-06
Chris pose la question :
How many different number combinations are there for the social security system? We know that there are only a certain ammount of combinations available and the current US population is at 281 million. The number system should include as many nine digit combinations as possible using the numbers 0-9 in multiple different spots.
Penny Nom lui répond.
Express 5120 as a sum of consecutive numbers 2002-08-25
Todd pose la question :
Express 5120 as a sum of more than 1 consecutive number.
Leeanne Boehm lui répond.
As much greater than 47 as it is less than 105 2002-07-23
Joe pose la question :
To find the number in question you need to find the number exactly half-way between 47 and 105. Another way of thinking about the number that is half-way between two other numbers is as the average of the two numbers given.
Leenaan Boehm lui répond.
Place six numbers around a triangle 2002-07-19
Monika pose la question :
I need to place six numbers around a triangle, as such that one number is on each corner, and one number on each side making three numbers in one line, adding up to two hundred exactly. The number I have to use are, 40, 50, 60, 70, 80, 90. I have to use each one once, there for, each number cannot be repeated.
Claude Tardif and Chris Fisher lui répond.
Can you make up a word that adds up to 1000000? 2002-07-19
Adrie pose la question :
My question is elementary and I am a student Please help me with the following conundrum, as it is driving me crazy.

'When the letters in the alphabet are given values from 1 to 26 for A to Z, can you make up a word that adds up to 1000000 ( one million )-used as a multiplication?'

Like 'cat' would be 3*1*20 = 60

Claude Tardif lui répond.
Square numbers 2002-05-01
Sally pose la question :
Is one considered a square number?
Penny Nom lui répond.
a+b=10 and ab=40 2002-04-27
April pose la question :
What two numbers add to ten and multiply to forty?

(a+b=10, a*b=40)

I think the answer includes radicals and/or imaginary numbers.


Penny Nom lui répond.
Triangular numbers 2002-04-26
Anika pose la question :
Can you please tell me what a triangular number is?
Penny Nom lui répond.
An Olympiad problem 2002-04-23
Aurora pose la question :
I wrote a mathematical Olympiad the other day and there was one question that I could not work out.

The question was If ab = 1, bc =2, cd =3, de =4 and ea = 6, what does a + b + c + d +e =

It was a multiple choice and the answers were : 43/6; 47/6; 49/6; 53/6; 61/6 The correct answer was 61/6, but how can do you get to that answer?


Anrei Volodin lui répond.
How many different account numbers are possible? 2002-04-18
Andrene pose la question :
how many different account numbers are possible if the account numbers consist of a letter of the alphabet, followed by five numerical digits, and followed by another letter.
Penny Nom lui répond.
The square root of i 2002-03-14
Arlene pose la question :
what is the square root of i, if i=x+yi?

what is the square root of 1-i? i'm getting problems like these in which I do not understand.


Harley Weston lui répond.
Mixed numbers, whole numbers and improper fractions which are less than 2. 2002-03-03
Dawn pose la question :
Change each mixed number to a whole number and an improper fraction which is less than 2. 3 1/2 = 2 /
Penny Nom lui répond.
Alfredos house number 2002-02-21
Aunt Patty pose la question :
Alfredos house number is between 20 and 35. The sum of the digits is less than 5. If you subtract 1 from it you would get a multiple of 3. If you add three, you get a multiple of 5. What is Alfredos house number.
Penny Nom lui répond.
Shipping charges 2002-02-19
Sohail pose la question :
A mail order company charges a fixed fee for shipping merchandise that weighs 1 pound or less, plus an additional fee for each pound over 1 pound. If there shipping charge for 5 pounds is $4.80, and for 12 pounds is $6.20, find the fixed fee and the additional fee.
Penny Nom lui répond.
A farmer takes 100 animals to the market to sell. 2002-02-19
Ann pose la question :
A farmer takes 100 animals to the market to sell. All are sold and $100 is earned. The prices were $5.00 per cow, $1.00 per sheep and $.05 per pig. At least one of each kind of animal was sold. How many of each kind of animal did the farmer take to market?
Penny Nom lui répond.
Square arrangements of clovers 2002-02-03
Cassie pose la question :
Clyde had a strange fascination with numbers. One day he decided to mount his 4 leaf clover collection in groups of square numbers.he took a long piece of butcher paper and glue and began this arduous task.There was 1 clover,4 clovers.and then 9 clovers in the third set. what would be in the seventh set of 4 leaf clovers?
Penny Nom lui répond.
Making 24 2002-01-17
Renee pose la question :
My 4th grade daughter and I need to find a simple math sentence using 5, 5, 3, & 7 to equal 24. You can add, subtract, divide or multiple.
Penny Nom lui répond.
What is a group of three numbers called in a large number? 2002-01-15
Laurie pose la question :
What is a group of three numbers called in a large number?

My son has this question on his 4th grade worksheet. I've taught middle school math for 7 years and nver heard of it.


Paul Betts lui répond.
Why do integers have the symbol of Z? 2002-01-11
Stephanie pose la question :
Why do integers have the symbol of Z? Natural numbers have the symbol of N, whole numbers have the symbol of W. Our professor has asked us to find out why the letter Z is represented for integers.
Penny Nom lui répond.
Some functions without numbers 2001-11-16
A student pose la question :
I have a worksheet that is about functions. It doesn't only use numbers. I need help to figure out the function and the solution to how the answer is solved.
Claude Tardif lui répond.
Numbering pages 2001-11-15
Lucy pose la question :
A printer uses 1008 digits to number the pages of a book. How many pages are there in the book?
Penny Nom lui répond.
Negative numbers 2001-11-15
Jewel pose la question :
If zero means nothing or an empty set, then how can a number be less than zero, as in negative number? My understanding of negative numbers is distance from a set point in a given direction. Thus having a negative and positive side of a line is arbritary and is related to displacement rather than value. Am I in error?
Penny Nom lui répond.
Squares of negative numbers 2001-11-03
Susana pose la question :
I wanted to know if I can square a negative number..?
Leeanne Boehm lui répond.
Squares of one digit numbers 2001-10-15
Needa pose la question :
What two two-digit numbers are each equal to their right-most digit squared?
Penny Nom lui répond.
eix = cosx + isinx 2001-10-10
Peter pose la question :
Given: eix = cosx + isinx
  1. substitute -x for x to find e-ix, simplifying your answer

  2. use the given and part a to find an identity for cosx making no reference to trig functions

  3. find an identity for sinx
  4. .
  5. .

Penny Nom lui répond.
Six nines 2001-10-09
A mom pose la question :
My middle schooler (sixth) has to calculate the integers 0-20 using only 6 nines. We have done all but the integer 14. He can not use decimals or double the nine like 99 or 19. the fraction 9/9 is okay. Keep in mind of course the order of operations.
Claude Tardif lui répond.
Building numbers 2001-10-02
Dorothy pose la question :
Using only the numbers 1, 2. 3 & 4 and the order of operations make problems that have the answers of 1 to 50. Use each digit only once per problem. Use +, -, x, (), exponents or use 2 digits to form a 2 digit number Example 1,2=21 or 12. No Division.

We have been able to do all the problems except finding the answers of 7, 22, 31, 34, 42, 43 and 50.


Claude Tardif lui répond.
What are the two numbers? 2001-09-24
A student pose la question :
What are the two numbers.
  1. the sum of two numbers is 15. 6 times the smaller number is 4 times the larger number .find the two numbers.
  2. the sum of the two numbers is 28. 2 times the larger number is 5 times the smaller number minus 7. find the two numbers.

Claude Tardif lui répond.
Integers 2001-09-09
A student pose la question :
I am a Grade 9 student from Ontario Canada who has never understood how to do integers very well. I was wondering if you would be able to either send me some sites that will tell me how to calculate them in a simple way or if you would be able to write me back and give me some pointers.
Leeanne Boehm lui répond.
I have 6 digits. 2001-09-06
Kristan pose la question :
I have 6 digits. My hundred thousands is 1 less than my 1 digit, 8 greater than my thousands digit, twice my tens digit, and 4 times my hundreds digit. My ten thousands digit is 0. What # am I?
Penny Nom lui répond.
Generating uniquie 2 digit numbers 2001-07-16
Anthony pose la question :
What methodology can I use to generate uniquie 2 digit numbers from unique 7 digit numbers. Of'course the total number of 7-digit numbers is not greater than 99.
Penny Nom lui répond.
6,000,000,000,000,000,000 2001-07-12
James pose la question :
How would I express 6,000,000,000,000,000,000 in words?
Leeanne Boehm lui répond.
Some irrational numbers 2001-07-03
Kellie pose la question :
  1. Is it possible to write pi as a fraction?

  2. Is it possible to write the square root of 2 as a fraction.. Explain why?

Harley Weston lui répond.
A calculation with 6 numbers 2001-06-16
Edwin pose la question :
I'm asked to come with, and program (in Ansi -C) an algorithm that calculates all the possible results of a calculation with 6 numbers and one result. For example: I want all calculations with the numbers 3, 3, 8, 8, 2, 9, and with a result of 786. all numbers may be used once, arithmetical operations allowed are + - / *, fractions are not allowed. The problem here is what is a fast method to do this (i.e. what's algorithm that can to this).
Claude Tardif lui répond.
Four digit numbers from 1,2,3,4,5,6,7,8 2001-05-31
Katie pose la question :
Okay.......here's my question...how many different ways can you form a four digit number out of these digits..1,2,3,4,5,6,7,8? This is how my teacher said to do this.... 8*8, 7*7,6*6,5*5,4*4,3*3,2*2,1*1. then she said to add up the products and to multiply by 7. is this correct, and if not how can you figure this out.
Penny Nom and Claude Tardif lui répond.
Three goldfish 2001-05-30
Nathan pose la question :
A man has three goldfish. When the youngest goldfish was born, the oldest fish was three times the middle fish's age. Nine years ago the oldest fish's age was the sum of the two other fish's ages. How old are the three goldfish?
Penny Nom lui répond.
Harmonic numbers 2001-05-23
Leslie pose la question :
The harmonic numbers Hk, k = 1,2,3.....are defined by Hk = 1 + 1/2 + 1/3....1/k

I am trying to prove by mathematical induction:

H2n >= 1 + n/2 , whenever n is a nonnegative integer.

H8 = H23 >= 1 + 3/2

Can you help?


Harley Weston lui répond.
Dividing fractions 2001-05-09
Rina pose la question :
I just wanted to ask if you could help me in math. See I'm having a test soon and its on Dividing Fractions and I just don't get it. My math teacher says that I'll be just fine but I failed my math quiz. I went to ask eric but they could help me so they told me to go to you. So here I am asking you if you could help me.
Penny Nom lui répond.
Neither prime nor composite 2001-05-08
Marc pose la question :
what 2 numbers are neither prime nor combitative?
Penny Nom lui répond.
24 from 2, 2, 14, and 21 2001-05-08
Kimberly pose la question :
using the numbers 2, 2, 14, and 21. how can you get 24 using + - X or division
Claude Tardif lui répond.
The number of occurrences of 0 is __, of 1 is __, of... 2001-04-17
Martyanne pose la question :
"In this sentence the number of occurrences of 0 is __, of 1 is __, of 2 is __, of 3 is __, of 4 is __, of 5 is __, of 6 is __, if 7 is __, of 8 is __, of 9 is __."

Each blank is to be filled with a numeral of one or more digits, written in decimal notation.

Claude Tardif lui répond.
Cube roots on a calculator 2001-03-24
Will pose la question :
Hi my name is Will.I have a question about the calculator TI-83 Plus or the use of a scientific calculator. It is about using them to turn a cubed root to a decimal. When using that for a rational and irrational number. Rational numbers is a number that terminates or repeates. A irrational number goes on and on and uses ... (like pie). Like when you type the cube root of 8 it gives you 2, and that is a rational number. The squre root of 2 is 1.41421356... that is irrational. So why is it when you put the cube root of 16 in the calculator it says 2.5198421 that is rational it terminates at the ninth digit and my float is set for the tenth? But why when you do it by hand you get the 2 times the cube root of 2 and that is irrational? Why is that?
Judi McDonald lui répond.
Fibonacci 2001-02-28
Shona pose la question :
We have been doing pattern finding with him, talking to a friend he mentioned the "Fibbinacci Series" ?, while I have tried to find a bit about it, how works etc.. what it is about, I have not really found out much, what I have I feel is way beyond him, but am still curious to know the basics of it myself. Would you be able to tell me in laymans language. Would be very much appreciated.
Claude Tardif lui répond.
T-numbers an T-totals 2001-02-27
Mr. M. Wiseman pose la question :
Investigate the relationship between the T-total and the T-number.
Penny Nom lui répond.
Find the two numbers 2001-02-21
A grade 6 student pose la question :
Find the two numbers:
5 is a common factor;
100 is a common multiple;
neither number is a mulitple of the other;
both are even.

Penny Nom lui répond.
Multiplying one and two digit numbers 2001-02-11
Marty pose la question :
When multiplying do you put the number you are multiplying by on the top or the bottom. Example if you are multiplying by 6's would you write the problem 6
x 2
or 2
x 6
Maybe it just doesn't matter. But when you get to double digits, the double digit always goes on the top?

Penny Nom lui répond.
23 + 17 * 2 / 8 + 11 = 13 2001-02-02
David pose la question :
My child is working through basic operations, and gave this problem:

23 + 17 * 2 / 8 + 11 = 13

The teacher wants the parenthesis to be put in the right places to come up with 13.
Claude Tardif lui répond.

Subsets of the natural numbers 2001-01-30
Christina pose la question :
How do I explain why the set of natural numbers (N) cannot be equivalent to one of its finite subsets?
Penny Nom lui répond.
14, with 5 ODD numbers 2001-01-23
Robyn Ernst pose la question :
How can you get 14, with 5 ODD numbers?
Claude Tardif lui répond.
More repeating decimals 2001-01-17
Alan pose la question :
I am neither a math teacher nor a student, but I hope you will consider my question anyway. I recently was discussing repeated decimals with a friend, and went on the web to find out more about a pattern was looking for years ago. In doing so I came upon your correspondence on repeating decimals.
.
.
.

Chris Fisher and Penny Nom lui répond.
Arranging some numbers 2001-01-14
Denise pose la question :
Arrange the following numerals in two groups of four each so that each group will add to give the same sum. 1 2 3 4 5 7 8 9
Penny Nom anf Claude Tardif lui répond.
8, 4, 2, 18, 1, 9 2001-01-11
Neyra pose la question :
Place each number below in one of the blanks to create the most meaningful and realistic story possible.
Claude Tardif lui répond.
The sum of all ten-digit numbers. 2001-01-10
John pose la question :
Compute the sum of all ten-digit numbers. Can you determine the sum of all n -digit numbers?
Sukanta Pati lui répond.
Two questions 2001-01-04
Justine pose la question :
first: A diagonal of a rectangle is the square root of 13 meters and the area of the rectangle is 6 meters squared. What are the dimensions of the rectangle?

last: Find the ordered triple of nonzero digits (A,B,C) for which (AA)squared=BBCC where AA and BBCC are respectively, two-digit and four-digit base-ten numerals.


Penny Nom lui répond.
Why the change from commas between every 3rd digit, to spaces instead? 2001-01-03
Clarende Duby pose la question :
Why the change from commas between every 3rd digit, to spaces instead? For example: Three million three hundred thirty-three thousand three hundred thirty-three written like: 3 333 333 instead of 3,333,333? Does it have something to do with standardizing mathematical notation worldwide?
Harley Weston lui répond.
Polite numbers 2001-01-02
A student pose la question :
A positive whole number, greater than 1, is said to be a polite number if it can be expressed as a sum of consecutive whole numbers

Find all the polite numbers less than 21. Find a pattern in the numbers that are not polite. Explain why every odd number greater than one must be polite.


Penny Nom lui répond.
Scientific notation 2000-12-12
Guy pose la question :
How do I express 1.000.000 in scientific notation?
Harley Weston lui répond.
Two more problems 2000-12-01
Michael pose la question :
first question:
The mean of three test grades is 74. What must a fourth grade be to increase the mean to 78?

final question:
The product of two consecutive odd positive integers added to their sum is 119. What are these two integers?


Claude Tardif lui répond.
A complex calculation 2000-11-24
Angie pose la question :
Multiply (3-2i)2=32-2(3)(2i)+(2i)2
Penny Nom lui répond.
Perfect numbers 2000-10-31
A student pose la question :
I was wondering if you could help me answer a question my pre-algebra teacher asked in class the other day. He asked if we knew what the perfect numbers where.

He told us the first number is 6 the second number is 28 but the third he did not tell us. Do you know what the third perfect number is?
Paul Betts and Chris Fisher lui répond.

Divisibility by 9 2000-10-24
Kelera pose la question :
If the sum of the digits of a number is divisible by 9, then the number itself it divisible by 9. Why is that? How do you explain this?
Penny Nom lui répond.
Repeating decimals 2000-10-06
Mary O'Sullivan pose la question :
I understand that with repeating decimals (those with a pattern), the number of digits repeated is put into fraction form with the same number of 9's

ex. 0.4444 = 4/9 0.145145145 = 145/999

How can I explain why this is so? Some inquisitive 8th graders are anxious to find out!
Penny Nom and Walter Whiteley lui répond.

Rounding 2000-09-14
MaryAnn pose la question :
How many whole numbers rounded to the hundreds place, round to 400? Can you also explain why?
Penny Nom lui répond.
Rational Numbers 2000-09-14
Josh Kuhar pose la question :
How can you tell a number is rational?
Harley Weston lui répond.
4-digit numbers using 7, 8, 9 and 0 2000-09-13
Ryan pose la question :
Could you please help me with the following problem, How many different 4-digit numbers are there that use each of the digits 7,8,9,and 0? Please list them.
Penny Nom lui répond.
A number problem 2000-09-02
Lalita Balu pose la question :
Can you arrange 1234567890 to add up to 100?
Penny Nom lui répond.
Central Limit Theorem and Law of Large Numbers 2000-06-26
Jonathan Yam pose la question :
The Central limit Theorem states that when sample size tends to infinity, the sample mean will be normally distributed. The Law of Large Number states that when sample size tends to infinity, the sample mean equals to population mean. Is the two statements contradictory?
Paul Betts and Harley Weston lui répond.
Names for large numbers 2000-06-16
Mireille Carthier pose la question :
I am trying to find the common names (in English and French) for the big numbers as follows:
English French
106 million million
109 billion milliard?
1012 trillion billion?
1015 ?? trillion?

Thank you very much


Claude Tardif and Harley Weston lui répond.
Percents with mixed numbers 2000-06-04
Julie pose la question :
How would you find the percent of 33 1/3 out of 90 by using fraction settup!I can do whole numbers but not mixed. This is how to do whole numbers say it was 33 of 90

33 x 90 = 2970
100 x 1 = 100
2970 divided by 100
the answer is 29.7

so how do you do it with a mixed number


Penny Nom lui répond.
Powers of i 2000-05-24
Paul Fieldhouse pose la question :
What is the result of raising i to the googol power? is there a rule or pattern to raising i by increasing powers of 10?
Penny Nom lui répond.
The square root of -1 2000-05-19
Gary pose la question :
i am not a student i am just some one that heard something and i can't be sure on the answer...my ? is what is the square root of -1? i think it is -1 but not sure can you let me know please thank you
Harley Weston lui répond.
Natural numbers 2000-04-12
Sara pose la question :
What is a natural number???
Penny Nom lui répond.
root(-1)* root(-1) 2000-03-20
Michael Moran pose la question :
i squared = -1

but

i squared = root(-1)* root(-1)
= root( -1*-1)
= root(1)
= 1
-1 doesn't = 1

can you help me with my question


Claude Tardif lui répond.
Six digit numbers using 1,2,5,6,7, and 9 2000-03-20
Rachel pose la question :
How many different six-digit numbers can you make using the digits 1,2,5,6,7, and 9? How many of these six digit numbers are divisible by six?
Claude Tardif and Denis Hanson lui répond.
Rationals, irrationals and integers 2000-03-14
Erin McKeon pose la question :
Why does the letter J represent the set of integers, the letter Q represent a set of rational numbers and the letter P represent a set of irrational numbers? What do each of these letters stand for?
Harley Weston lui répond.
Triangular numbers and square numbers 2000-03-02
Emily McCallum pose la question :
Would someone find me some math activities using triangular numbers and square number. Especially acitivities that actually form the shapes. They need to be at the 5th grade or 6th grade level. But you can not find anything that can be tauht at these level, that fine. I just need to be able to teach this new subject or kind of math to my kids.
Rick Seaman and Penny Nom lui répond.
More on googols 2000-02-23
Kevin Brennan pose la question :
Could you please give me a couple of examples of when you MATH guys would use a google. Do youu measure, like, galaxies with it ? Or was it created to keep track of Mr Gate's bank account.
Penny Nom lui répond.
Compatible numbers 2000-01-27
Angie pose la question :
Use compatible numbers to estimate each product and quotient.
23*1/2
1/3*11

Penny Nom lui répond.
Factors 2000-01-27
Pam pose la question :
My son [age 9, 4th grade] has a math problem with factors. I need to know how to do the problem that involves the factor 24. I'm not even sure where to start.
Harley Weston lui répond.
Phone Number Possibilities 2000-01-09
Hamilton Weston pose la question :
How do phone companies arrive at the possible number of phone numbers that can be generated for customers?
Harley Weston lui répond.
Fibonacci Numbers 1999-12-15
Gary Nelb pose la question :
I'm doing a project on fibonacci numbers and I'm using different starting values and finding out if different starting values to see whether or not the ratios still get closer to phi. I was wondering, what numbers should I use. Should I use two of the same # like 2 and 2, or numbers like 1 and 2, or even something totally different.
Denis Hanson lui répond.
Bases other than 10 1999-12-06
Garret Magin pose la question :
We are doing a lesson on numbers of other bases than 10. We are working with binary, octal, and Hexadecimal. I was wondering what is used to represent number of different bases other then 16? Does it just continue on with the alphabet and if so what happens when you get to Z. It would be a help if you could answer this because it is really bugging me. And none of the math teachers at my school could let me know.
Claude Tardif and Patrick Maidorn lui répond.
Whole numbers 1999-12-02
Patty pose la question :
I'm in a 5th grade class, please explaine what whole numbers are.....
Penny Nom lui répond.
Critical numbers 1999-11-17
Sarah pose la question :
My question is this:

If f(x)=2x3+4x2-9x+8

and the derivative is f'(x)=6x2+8x-9,

how do I find the critical points if f'(x) is not factorable??
Penny Nom lui répond.

Number sequences 1999-11-11
Justin Schessler pose la question :
I CAN NOT figure out this sequence or how we continue this sequence...

3,2,6,5,15,14,___,___,___
Chris Fisher and Harley Weston lui répond.

Nines and ones 1999-11-05
Greg Miller pose la question :
Using the numbers 1, 1, 9, 9, only once each, how can I create an expression that equals 10 using only the basic functions of addition, subtraction, multiplication, division, and/or parentheses?
Claude Tardif lui répond.
Three numbers 1999-11-03
Walter pose la question :
Three whole numbers , when added two at a time,yield the sums 75, 111, and 128.

Find the smallest of these three whole #'s.
Penny Nom lui répond.

Find the missing numbers 1999-10-12
Bonnie pose la question :
I must find the missing numbers and function rules

noutput
710
1220
3940
2430
16b
87c

we are totally confused can someone help
Denis Hanson lui répond.

A sum of two squares 1999-10-08
Marksmen pose la question :
what is the smallest whole number that can be written two ways as a sum of two different perfect squares? i.e.11squared plus 3 squared is 121+ 9=130 and7 squared + 9squared=49 +81= 130. Are there any smaller? I am stumped!
Claude Tardif lui répond.
Numbers with the digit 2 in 1...1000 1999-09-20
Jessica pose la question :
Is there a trick to finding out how many numbers containing the digit two is there from 1 to 1000?
Walter Whiteley lui répond.
Evens and odds 1999-05-04
Emily Hays pose la question :
My name is Emily Hays, I am in the 6th grade and my teacher gave me some extra probelms to see if I could solve them. She can't find the KEY and I can't figure out the probelm so she suggested I e-mail you guys to see if you could help us. Here's the Problem:

The numbers 1,2,3...,1999 are written on the blackboard in the classroom. Evertime the teacher enters the room he chooses two numbers on the blackboard, say a & b, with a> or equal to b, then he erases them and writes the difference a-b somewhere on the blackboard. After this procedure is carried on 1998 times, there will be only one number left on the blackboard. Prove the last remaining number must be even.

I hope you can help us!
Chris Fisher and Harley Weston lui répond.

Large Numbers 1999-04-05
CK pose la question :
Hello. I need a lesson plan for grades 4 to 7 dealing with the concept of large numbers. Specifically, how to teach scientific notation. The lesson plan has to deal with the following: the size of the universe is so huge that is is almost beyond the ability of the mind to comprehend. One way is to measure...
Chris Fisher lui répond.
T-shirts 1999-04-04
Valerie pose la question :
At a discount T-shirt shop the manager marked a special batch of T-shirts that originally sold for $2.00. When all the shirts sold the shop had collected $603.77.

How many shirts did they sell and what was the price per shirt?
Jack LeSage and Penny Nom lui répond.

Complex numbers/polar coordinates 1999-03-25
Kate Cegelis pose la question :
What is the relationship between complex numbers and polar coordinates?
Harley Weston lui répond.
The square root of two is never supposed to end 1999-03-06
a wondering student pose la question :
i am algebra II and am in the 9th grade. today we were talking about rational and irrational numbers. When we were talking about square roots my friend and i were talking and we thought of something. if you have a square with sides of length one then the diagonal of the square is the square root of 2. Now the square root of two is never supposed to end. But the diagonal of the square ends so therefore doesn't the square root of 2 end. our math teacher did not really answer our question because it was not in the lesson plan and not to many people would see where we were coming from. the answer is really bugging me and i would like to have your input.
Jack LeSage and Penny Nom lui répond.
The Board Problem 1999-02-15
Avery pose la question :
Mr. Avery has 3-foot boards and 4-foot boards. If he puts the 3-foot boards in a line, they have the same length as the 4-foot boards put in a line. Altogether he has between 16 and 25 boards. How many 3-foot boards does he have?
Jack LeSage and Penny Nom lui répond.
Magic Squares 1999-02-11
Katie Powell pose la question :
My name is Katie Powell. I'm in the 7th grade, taking Algebra. I live in Houston, Texas. My problem is this:

"Use the numbers 1-9 to fill in the boxes so that you get the same sum when you add vertically, horizontally or diagonally."

The boxes are formed like a tic-tac-toe -- with 9 boxes -- 3 rows and 3 columns.

Can you help?
Jack LeSage lui répond.

Grade 4 curriculum 1999-02-09
John pose la question :
My daughter is in grade 4 and her math teacher has asked her to write out the number 1 to 10,000. She is currently at 4567 and is sick of it. When I complained that this was not a usefull exercise she said that the Ministry of Education curriculum requires this and gave a photocopy of the page. It states: "read and write whole number to 10,000 in standard, expanded, and written forms (eg., 9367=9000+300+60+7 = nine thousand three hundred sixty-seven). Has the teacher interpreted this the wrong way?

Thanks John
PS. Do you have a web address for Ontario Math and Science Teachers organization?
Jack LeSage lui répond.

Cannonballs 1999-01-27
Roger King pose la question :
How many cannonballs can be stacked in a triangular pyramid?
Penny Nom lui répond.
Absolute value of i 1999-01-06
Wayne Bagley pose la question :
I would like to know what is the absolute value of i. I need an answer suitable for the secondary level.
Harley Weston lui répond.
Place Value 1999-01-05
Reg pose la question :
I work in a Section 27 classroom with various grades. I am looking for assistance on how to deliver a lesson on place value:decimal numbers keeping in mind that they need to see it (visual) to comprehend. I am looking for grade 4-6 level.
Jack LeSage lui répond.
Complex numbers and the quadratic formula 1998-12-25
Richard Peter pose la question :
My age is 16, and my name is Richard. My question relates to the topic complex numbers & the quadratic formula.

I would like to know how to solve quadratic equations in which the discriminant is less than 0 (i.e. we get two complex solutions to the quadratic)

3x2+2x+5 = 0

and how mathematicians like euler contributed to this field. If it would be possible I would also like to know how this type of quadratic equations can be graphed?
Harley Weston lui répond.

Complex Numbers 1998-12-23
Wayne Bagley pose la question :
I would like to know what is the square root of i , and i squared? I am looking for a response appropriate for secondary level students.
Harley Weston lui répond.
6.99999... = ? 1998-12-05
Tom pose la question :
I have had a rather heated arguement with my students. Please settle this for me.

Solve <,>, =
6.99999... __ 7

Thank you.
Penny Nom lui répond.

Triangular Numbers 1998-10-30
Matt pose la question :
i would like to know about triangular numbers and it history i would also like to know about the history of prime numbers thank you
Chris Fisher lui répond.
A Number Trick 1998-10-26
Brenda Meagher pose la question :
I'm a 37 year old mother and i have returned to school for personal reasons. I was faced with this question.
Choose any number less than 10, multiply it by 8547, then take your answer and multiply it by 13. I chose the number 8.

8547
x 8
--------
68376

and

68376
x 13
---------
205128
68376
---------
888888

Any number chosen from 1 to 9 will result in the same form of answer.If the number two is used you will end up with 222,222.etc So my question is, is there another number that will results in the answers.
Harley Weston lui répond.

Adding Fractions 1998-10-15
Lindsay pose la question :
please help me!!!!
Harley Weston lui répond.
Composite Numbers 1998-10-07
Greg Murphy pose la question :
What type of composite number has an odd number of factors?
Harley Weston lui répond.
Scientific Notation 1998-10-07
R. Pierce pose la question :
Help! How do you express the following numbers in scientific(exponential)notation:0.00036 and 2998000000. I understand how to solve them if it was an equation, but this is confusing me. Please help me answer my questions. Thank you.
Harley Weston lui répond.
Rational and Irrational Numbers 1998-09-19
Ri pose la question :
I am trying to explain rational & irrational numbers to my niece who is grade 7 and am having difficulties. Could you please explain the difference between rational & irrational numbers.

Thank you
Ri
Penny Nom lui répond.

Rational Numbers 1998-09-05
Kristin Mckenzie pose la question :
Iam a secondary student with a math question I don't know how to do.
This is the question:

Explain whether each decimal number represents a rational number.

(a) 0.16

(b) 0.12

(c) -3.125

(d) -0.27

(e) -0,212 112 111 2 ....

(f) 0. 457 92

If it wouldn't be a problem i would really appreciate it if you sent back the instructions on how to do the question.

My name is Kristin Mckenzie and my return e-mail is lexus1999@hotmail.com

Thank-you for you time
Jack LeSage and Harley Weston lui répond.

Two Problems 1998-07-28
James Pulver pose la question :
How do you solve these problem? If log abc=16 and log ac=12 , find b. (The logs are log base 10.)
and
If a and b are real numbers, i^2 = -1 and (a+b)+5i=9+ai what is the value of b?

Jack LeSage lui répond.
Social Security Numbers 1998-07-07
Valerie Lucas pose la question :
How many social security numbers are possible?

the answer is 10^9

why isn't the answer 9^9?
Harley Weston lui répond.

Roman Numerals 1998-02-05
Dawn Hilgenberg pose la question :
How would I read a Roman Numeral such as LXII? Does the L in front make the X equal a lesser amount? We have been discussing how to read these and I can find no information about how to read an extended Roman Numeral problem. Help!!
Harley Weston lui répond.
Divisors of 6n 1998-01-26
James pose la question :
If n is a positive integer such that 2n has 28 positive divisors and 3n has 30 positive divisors, then how many positive divisors does 6n have?
(a) 32 (b) 34 (c) 35 (d) 36 (e) 38

Penny Nom and Haragauri Gupta lui répond.
The two-digit numbers from 10-99 1998-01-21
Alexis Riddle pose la question :
My name is Alexis. I'm a student in 8th grade and I'm taking Algebra I Honors. My question is: In a string of numbers, two adjacent digits are considered as a two-digit number. For instance, the string 11012 contains the numbers 10, 11, and 12. What is the number of digits in the smallest string that contains all of the two-digit numbers from 10-99? Please help and thank you for any assistance you can give me.
Penny Nom lui répond.
Find the Numbers. 1997-12-28
Sandy C pose la question :
The sum of the squares of two numbers is 53. Twice the greater, minus the lesser is 11. Find the numbers.
Penny Nom lui répond.
Driving me CRAZY. 1997-11-17
Billy Tran pose la question :
Write a two digit number (both digits different and neither equal to zero) and then express the same number but now composed of the same two digits in reverse order with some mathematical symbol (+, -, *, /, roots, !, or exponents). for example: 25 = 5^2. Now give me another.

Hope you get this, cause its driving me CRAZY
Penny Nom lui répond.

Negative Primes. 1997-11-10
Leah Zucker and Paul Michael pose la question :
I have received the following question via e-mail from my granddaughter: "Can negative numbers (like -7) be prime? If not, why not?"
Chris Fisher lui répond.
Multiplying imaginary numbers. 1997-11-03
Jim Catton pose la question :
Here is the question:

(square root -2) x (square root -8)

My algebra suggests two possibilities .
.
.

Walter Whiteley, Chris Fisfer and Harley Weston lui répond.

Billions and more! 1997-09-15
Mahabir B. Gupta pose la question :
I would like to know how you americans write the number 1 billion.

Do you say "One thousand million"..can you answer by giving me examples?

1,000,000----> 1 million
1,000,000,000---->1 billion

Why is it that in spanish it is different?
Penny Nom lui répond.

Triangular Numbers. 1997-09-08
Rachel pose la question :
Show visually that the square number 16 is the sum of two triangular numbers. Which two?
Penny Nom lui répond.
Prime Numbers 1997-09-04
Jack Bedard pose la question :
Hi. I have a real stupid question

What are prime numbers?

My name is Jack Bedard and I am in grade 5.
Penny Nom lui répond.

Repeating Decimals 1997-01-24
Grant Reed pose la question :
Is there a way to tell that the repeating decimal for 1/17 has no more than 16 repeating digits?
Penny Nom lui répond.
Term definitions 1996-09-18
M. Seltzer pose la question :
A 7th grade algebra student would like the definition of the terms proper factor and perfect number.
The Centralizer lui répond.
How do you find out if a number is a composite or prime? 1996-09-16
Trish Feenstra pose la question :
Hi my name is Trish Feenstra. In grade 8. How do you find out if a number is a composite or prime if it is big like 37 529? Reply soon got a test on Friday the 13th
Denis Hanson lui répond.
Thousands, millions, billions and trillions 1996-08-22
Blaine pose la question :
What are the names of the periods in groups of numbers like 123,456,789. I need to know the names of them for school. I already know the first ones: units thousands millions billions trillions.
Penny Nom and Diane Hanson lui répond.
Prime numbers 1996-02-22
Charles Hewitt pose la question :
I would apprecaite it if you could share with me any algorithm to determine whether a number is prime.
Penny Nom lui répond.
Complex numbers 1995-10-22
Jacquie pose la question :
Why should we teach complex numbers in high school?
Harley Weston lui répond.
 
Page
1/1

 

 


Centrale des maths reçoit une aide financière de l’Université de Regina et de The Pacific Institute for the Mathematical Sciences.

CMS
.

 

accueil centre de ressources accueil Société mathématique du Canada l'Université de Regina PIMS